Download as pdf or txt
Download as pdf or txt
You are on page 1of 42

‫اﻟﻠﻬﻢ اﺳﺘﻮدﻋﻚ ﻣﺎ ﻗﺮأت وﻣﺎ ﺣﻔﻈﺖ‬

‫اﺳﺌﻠﺔ وزارة اﻟﺼﺤﺔ ﻟﻼﻟﺘﺤﺎق‬ ‫وﺗﻌﻠﻤـﺖ ‪ ،‬ﻓـﺮده ﻟـﻲ ﻋﻨـﺪ ﺣﺎﺟﺘـﻲ اﻟﻴﻪ ‪,‬‬

‫ﺑﺒﺮﻧﺎﻣﺞ اﻻﻗﺎﻣﺔ واﻻﻣﺘﻴﺎز‬ ‫إﻧـــﻚ ﻋﻠﻰ ﻣﺎ ﺗﺸﺎء ﻗﺪﻳﺮ‬

‫) ‪( 2018 -2013‬‬
‫اﺧﻮﻛﻢ اﻟﺪﻛﺘﻮر ‪ :‬اﺳﺎﻣـﻪ اﻟﺨﺰاﻋﻠـﻪ‬
‫ﻗﻤﺖ ﺑﺎﻋﺎدة ﺟﻤﻊ وﺗﻨﺴﻴﻖ‬ ‫ﺑﻔﻀﻞ ا‬
‫‪0799430239‬‬
‫اﺳﺌﻠﺔ اﻟﺪورات اﻟﺴﺎﺑﻘﺔ‪ ,‬اﻟﺼﺎدرة ﻣﻦ‬ ‫ﻃـــﺐ ﻋـــﺎم وﺟﺮاﺣــﻪ‬
‫وزارة اﻟﺼﺤﺔ ﻻﻣﺘﺤﺎﻧـــﺎت اﻻﻗﺎﻣـــﺔ‬

‫) ‪-2013‬‬ ‫واﻻﻣﺘﻴــﺎز ﻟﻼﻋـــﻮام‬

‫‪ , ( 2018‬ﺑﻌﺪ ﺟﻬـــﺪ وﻋﻨــﺎء ‪ ,‬وﺣﻠﻬﺎ‬

‫ﺑﺎﻻﺳﺘﻌﺎﻧﺔ ﺑﺎﻟﻤﺮاﺟﻊ واﻟﻜﺘﺐ اﻟﻤﺨﺘﻠﻔﺔ‬

‫ﻟﺘﻮﺧﻲ اﻟﺪﻗﺔ ﻓﻲ ﻧﺸﺮ اﻟﻤﻌﻠﻮﻣﺔ‬

‫اﻟﺼﺤﻴﺤــﺔ ﻟﻠﺰﻣﻼء واﻟﺰﻣﻴﻼت ‪ ,‬ﻓﺈن )‬

‫‪ ,‬وان اﺳﺄت ﻓﻤﻦ‬ ‫اﺣﺴﻨﺖ ﻓﻤﻦ ا‬

‫ﻧﻔﺴﻲ ( ‪ ,‬واﻧﻲ اﻗـــﺪم ﻫﺬا اﻟﻌﻤﻞ ﺧﺎﻟﺼﺎ‬

‫ﺗﻌﺎﻟﻰ ‪ ,‬راﺟﻴﺎ اﻟﻤﻮﻟﻰ اﻟﻘﺒﻮل ‪,‬‬ ‫ﻟﻮﺟﻪ ا‬


‫*****‬
‫وان ﻳﻨﺎل اﻋﺠﺎﺑﻜﻢ ‪.‬‬
* Elderly came with sudden loss of vision in right

MEDICINE
eye with headache , investigation show high CRP
and high ESR . What is the diagnosis ?
A . Temporal arteritis .
B . SAH .
C . Migraine .
D . Herpes ophthalmicus .
E . Tension headache .

* Acute renal failure may be distinguished from

*****
chronic renal failure by which of the following ?
A . An increased Na excretion .
B . Left ventricular hypertrophy on ECG .
C . Hypophosphatemia .
D . Renal size on ultrasound scan .
E . Hyperkalemia .

* In a patient with haematemesis . The clinical

‫ﻧﺴﺨﺔ ﻣﺪﻗﻘﺔ‬ feature which suggests that peptic ulcer is


underlying cause is ?
A . Ascites
B . Drowsiness
C . History of jaundice
D . Splenomegaly
E . Tender epigastrium

* Diabetes insipidus is a deficiency of what


hormone of the following ?
A . Atrial natriuretic peptide . * A 40 years old man presents with cold
B . Vasopressin . intolerance and weight gain . Examination
C . Aldosterone . reveals goiter . The most likely finding related to
D . Insulin . central nervous system in this patient ?
E . Progesterone . A . Ataxia .
B . Delayed relaxation of ankle jerk .
* An old lady presents with history of fever and C . Hyperreflexia .
left sided chest pain for one month . Examination D . Hypotonia .
of respiratory system shows decreased stony E . Loss of sensations .
dull
percussion note an absent breath sounds on left * Clostridium difficile is a nosocomial infection .
side . Her chest X_Ray is likely Which of the following can be used to treat it ?
to reveal ? A . Broad spectrum antibiotics .
A . Collapse B . PPI .
B . Consolidation C . Probiotics .
C . Fibrosis D . Vancomycin or metronidazole .
D . Pleural effusion E . No treatment is required
E . Pneumothorax
* A 15 years old girl presents with history of
Which of the following is least likely to cause fever , bleeding from gums and pallor for last 15
symmetrical weakness of the lower limbs ? days . Her peripheral blood films shows
A . Acute transverse myelitis pancytopenia . The most important investigation
B . Glioblastoma multiforme is ?
C . Guillain barre syndrome A . Bone marrow examination
D . Myasthenia gravis B . Coombs test
E . Muscular dystrophy C . Reticulocytes count
D . Serum folic acid
E . Serum iron level
‫ اﺳﺎﻣﻪ اﻟﺨﺰاﻋﻠﻪ‬: ‫اﻟﺪﻛﺘﻮر‬ ‫اﺳﺌﻠــــــــﺔ اﻟﺒﺎﻃﻨﻴــــــﺔ‬
Page 2
A . Coarctation of aorta
* The most sensitive vaccine to heat is ? B . Renal artery stenosis
A . IPV C . Dissection of aorta
B . BCG vaccine D . Conn syndrome
C . Hepatitis B E . Diabetic nephropathy
D . OPV
E . Diphtheria * A patient with history of jaundice , pruritus and
clay coloured stool . Which of the following
* In a patient with history of shortness of breath , enzymes may be increased significantly ?
which of the following signs indicates left heart A . ALP
failure ? B . AST
A . Ascites . C . ALT
B . Basal crepitations . D . LDH
C . Dependent edema .
D . Engorged neck veins . * A patient presents with history of intermittent
E . Fourth heart sounds . fever , abdominal pain and headache . Blood
culture is positive for salmonella typhi . The
* In a patient with high grade fever , rigors and antibiotic of choice for this patient ?
Hepatomegaly . The most likely diagnosis is ? A . Ciprofloxacin
A . Carcinoma of liver B . Vancomycin
B . Liver abscess C . Metronidazole
C . Malaria D . Tetracycline
D . Right heart failure E . Gentamicin
E . Typhoid fever
* A 50 years old man is admitted in emergency
ward with MI . Any of the following medications
are used initially ?
A . Aspirin
B . Warfarin
C . Heparin
D . Streptokinase
* Which one of the following is a contraindication E . Clopidogrel
to thrombolysis ? * A 40 years old lady presents with history of
A . Pregnancy severe constant upper abdominal pain ssociated
B . Age over 75 years with vomiting , temperature is normal , and there
C . Background diabetic retinopathy is marked tenderness in epigastrium . Most seful
D . Asthma investigation for this patient is ?
E . The presence of atrial fibrillation A . Cardiac enzymes
B . ECG
* A young boy presents in outdoor with history of C . Gastroscopy
sudden painful swelling of joints after minor D . Liver function test
trauma . The most likely diagnosis is ? E . Serum amylase
A . DIC
B . ITP * A 67 years old man presents with bilateral ankle
C . Sickle cell disease edema and pitting . Which of the following is the
D . Hemophilia A most common cause ?
E . Glanzmann thrombasthenia A . Amlodipine
B . Bisoprolol
* CRP may be raised in all of the following except C . Ruptured of Baker cyst
A . DM D . DVT
B . Malignancy E . Cellulitis
C . Acute bacterial infection
D . Tuberculosis * A 50 year old smoker presents with history of
E . Autoimmune disease cough productive of mucoid sputum every winter
for last three years . The most likely diagnosis is
* In a young boy with HTN , examination CVS A . Bronchial asthma
reveals radio _ femoral delay . The most common B . Chronic bronchitis
cause of HTN in this patient is ? C . Bronchiectasis
‫ اﺳﺎﻣﻪ اﻟﺨﺰاﻋﻠﻪ‬: ‫اﻟﺪﻛﺘﻮر‬ ‫اﺳﺌﻠــــــــﺔ اﻟﺒﺎﻃﻨﻴــــــﺔ‬
Page 3
D . Bronchogenic carcinoma * A 20 year old Girl is taking anti tuberculosis
E . Pulmonary tuberculosis treatment , she presents in eye outdoor with
visual complaints . The most likely cause of her
* In a patient with history of muscle cramps and symptoms is side effects of ?
carpopedal spasm . Which of the following A . Isoniazid
serum electrolyte level is most likely to be B . Pyrazinamide
decreased ? C . Rifampicin
A . Calcium D . Ethambutol
B . Chloride E . Streptomycin
C . Magnesium
D . Potassium * A 40 years old man gives history of high grade
E . Sodium fever for last one week with cough productive
rusty sputum , breathing on right lower chest ,
* The anti diabetic agent of choice for 50 years CXR shows consolidation . The most likely
old obese lady with mild hyperglycemia is ? causative organism is ?
A . Chlorpropamide A . Anaerobic bacteria
B . Metformin B . Gram negative bacilli
C . Insulin C . Streptococcus pneumonia
D . Glibenclamide D . Staphylococcus aureus
E . Repaglinide E . Mycobacterium tuberculosi

* A patient of chronic diarrhea is having angular * In a patient with microscopic Hematuria , which
stomatitis and glossitis . The most likely cause of the following favours a glomerular source ?
of these signs is deficiency of ? A . Red cell casts and proteinuria .
A . Folic acid B . Blood at the beginning of the urinary stream .
B . Iron C . Blood at the end of the urinary stream .
C . Proteins D . Suprapupic pain .
D . Pyridoxine E . Abnormal urine cytology .
E . Thiamine
* An important physical sign of portal
hypertension in a patient of cirrhosis of liver is ?
A . Gynecomastia
B . Hepatomegaly
C . Palmer Erythema
* A 40 years old woman gives history of fever for D . Spider angioma
last three weeks accompanied by dry cough , E . Splenomegaly
night sweats and weight loss . Abdominal * An old patient presented in emergency ward
examination reveals hepatosplenomegaly , CXR with history of weakness of right side of body of
shows symmetrically distributed line nodules . rapid onest . The most helpful first line
The most likely diagnosis is ? management of this patient is ?
A . Pneumonia A . Cerebral angiography
B . Malaria B . Cerebrospinal fluid examination
C . Typhoid C . Computerised tomography scan brain
D . Chronic liver disease D . Fasting lipid profile
E . Miliary tuberculosis E . Nerve conduction study

* A 40 years old diabetic man presents with * A 50 years old man presents with Dysphagia .
history of sudden onest of pain in right loin which Which of the following characteristics suggests a
radiates towards the iliac fossa associated with benign strictures of esophagus ?
vomiting .On examination tenderness is present A . Anemia
in right loin . The most likely diagnosis is ? B . Weight loss
A . Appendicitis C . Cervical lymphadenopathy
B . Cholycystitis D . Hoarseness of voice
C . Diverticulitis E . Dysphagia worse for solids
D . Perforated peptic ulcer
E . Pyelonephritis * Which of the following is a cause of central
cyanosis ?
A . Exposure to cold
‫ اﺳﺎﻣﻪ اﻟﺨﺰاﻋﻠﻪ‬: ‫اﻟﺪﻛﺘﻮر‬ ‫اﺳﺌﻠــــــــﺔ اﻟﺒﺎﻃﻨﻴــــــﺔ‬
Page 4
B . Heart failure C . Coal mining
C . Raynauds phenomenon D . Low socioeconomic status
D . Right to left cardiac shunts E . Tobacco smoke
E . Shock
* Which one of the following statements about
* A young girl complains of nocturnal cough and paracetamol poisoning is not true ?
shortness of breath which disturbs her sleep . It A . 15 tablets may be a fatal overdose .
was diagnosed as bronchial asthma . B . Oral methionine may be useful in treatment .
Investigation to confirm this diagnosis is ? C . Paracetamol levels should be checked at 4
A . CXR hours post ingestion .
B . Eosinophil count D . If liver function tests are normal at 4 hours
C . Lung function test post ingestion , the liver has not been damaged .
D . Serum IgG level E . Activated charcoal should be given with
E . Sputum examination laxatives .

* Which of the following does not form part of * A 35 years old man presents in a clinic with
the initial approach to investigating a fever of history of chronic productive cough that is worse
unknown origin ? in the morning and that occurs because
A . Early CT scan of abdomen and chest changing the position of patient . Sputum is
B . Serial blood cultures off antibiotics copious and yellow . The most likely diagnosis in
C . Careful history and examination this patient is ?
D . Autoantibody screen A . Bronchial asthma
E . Trial of antibiotics B . Bronchiectasis
C . Bronchogenic carcinoma
* An epileptic girl is found to have gum D . Pulmonary tuberculosis
hypertrophy . Anti epileptic drug which she is E . Chronic bronchitis
most likely taking is ?
A . Carbamazepine * A 50 year old man presents in emergency ward
B . Gabapentin with central chest pain . On examination his
C . Lamotrigine blood pressure 90/60 and pulse 106 , and he is
D . Phenytoin pale and sweating profusely . The most likely
E . Sodium valproate diagnosis is ?
A . Pericarditis
B . MI
C . Esophagitis
D . Pleural effusion
* A patient presents with tachycardia , wild E . Pneumothorax
swings in blood pressure , headache , * The following are complication of Nephrotic
diaphoresis , altered mental status , and a sense syndrome with the exception of ?
of panic The most likely diagnosis is ? A . Acute renal failure .
A . Hyperthyroidism . B . Accelerated HTN .
B . Generalized panic disorder . C . Venous thrombosis .
C . Angina pectoris . D . Hypocalcemia .
D . Panic attack . E . Pneumococcal infection .
E . Pheochromocytoma .
* Which of the following is a recognized feature
* A 16 year old Girl presents with chronic of massive pulmonary embolism ?
diarrhea , which of the following features suggest A . Reduced plasma lactate levels .
that she has irritable bowel syndrome ? B . Increase in serum troponin levels .
A . Abdominal pain relieved by defecation . C . An arterial PH less than 7.2 .
B . Anemia . D . Blood gases show increased PCO2 on air .
C . Weight loss . E . Normal D _ dimer levels .
D . Nocturnal symptoms .
E . Blood in stools . * A patient with embolic stroke is taking
warfarin . The most useful laboratory test to
* The most common risk factor for COPD is ? monitoring effect of warfarin ?
A . Air pollution A . PT
B . Infection B . PTT
‫ اﺳﺎﻣﻪ اﻟﺨﺰاﻋﻠﻪ‬: ‫اﻟﺪﻛﺘﻮر‬ ‫اﺳﺌﻠــــــــﺔ اﻟﺒﺎﻃﻨﻴــــــﺔ‬
Page 5
C . Bleeding time depression , and T waves inversion on the ECG ,
D . Hematocrit there is no chest pain . In mean time what should
E . Platelet count you do ?
A . Stop Benzaepril and digoxin .
* A 68 year old patient with COPD was admitted B . Stop verapamil and lasix .
with a febrile exacerbation of his symptoms . C . Stop Nebivolol and lasix .
Laboratory showed arterial PH 7.21 , PCO2 60 D . Stop lasix and digoxin .
mmhg , HCO3 24 meq/L . Which is the underlying E . Stop verapamil and aspirin .
acid base disorder ?
A . Respiratory acidosis . * A 52 year old man comes to the emergency
B . Respiratory acidosis with metabolic department because he has had vomiting,
Alkalosis . nausea, and abdominal pain for the past 12
C . Respiratory acidosis with metabolic acidosis . hours . He says he attempted suicide . 3 days
D . Metabolic acidosis with respiratory alkalosis . ago by taking everything in the medicine cabinet .
E . Metabolic acidosis He was stuporous for approximately 12 hours
after the overdose . But felt better the following
* 35 years old man is admitted to the hospital day . At this time , he has jaundice and pain in the
and is undergo brain surgery . The patient is very RUQ . Which of the following drugs is most likely
anxious and worried of the upcoming surgery . to have caused the pain , vomiting and jaundice ?
He begins to hyperventilate and becomes dizzy A . Acetaminophen
and unconscious , the ABG reveal PH 7.61 , PCO2 B . Aspirin
22 mmhg , HCO3 24 meq/L . Most likely acid C . Cimetidine
base disorder is ? D . Diphenhydramine
A . Metabolic acidosis uncompensated . E . Triazolam
B . Respiratory Alkalosis compensated .
C . Respiratory Alkalosis uncompensated . * A patient present with weakness , nausea ,
D . Metabolic Alkalosis compensated . vomiting , weight loss and new skin pigmentation
E . Metabolic Alkalosis uncompensated Labs show Hyponatremia and Hyperkalemia ,
most likely diagnosis is ?
* Most common cause of Wallenberg syndrome A . Tinea versicolor
is occlusion of ? B . SIADH
A . Basilar artery . C . Addison disease
B . Anterior cerebral artery . D . Leprosy
C . Middle cerebral artery . E . Phenylketonuria
D . Posterior inferior cerebral artery .
E . Posterior cerebral artery

* A 70 year old man who is presented with 2


* A 5 years old boy gives history of swelling of weeks history of abdominal pain that has
body starting from face and more on getting up worsened over the last few days , on examination
in the morning . On examination pallor is absent the patient is jaundice with tenderness over the
jugular venous pressure is not raised . Signs of epigastric area , with smooth hepatomegaly and
Ascites and bilateral pleural effusion are found , shifting dullness , which of the following is the
the first line of investigation is ? cause of hepatomegaly ?
A . Blood urea level . A . IDA
B . CXR . B . Budd Chiari syndrome
C . Echocardiography . C . Ulcerative colitis
D . Liver function test . D . Crohn disease
E . Urine for albumin . E . Left sided heart failure

* An 70 year old man presents with palpitations . * In patient with microscopic Hematuria , which
He has history of MI in 1995 and was placed on of the following favours a glomerular source ?
nitroglycerin , Benzaepril , aspirin and verapamil . A . Red cell cast and protein in urine
In 2000 he was placed on digoxin after a B . Blood at the beginning of voiding
diagnosis of heart failure was made . Three C . Blood at the end of voiding
months ago he was placed on Nebivolol and lasix D . Suprapubic pain
( Furosemide ) . He presents with evidence of E . WBC in urine
premature ventricular contractions , ST
‫ اﺳﺎﻣﻪ اﻟﺨﺰاﻋﻠﻪ‬: ‫اﻟﺪﻛﺘﻮر‬ ‫اﺳﺌﻠــــــــﺔ اﻟﺒﺎﻃﻨﻴــــــﺔ‬
Page 6
* A 25 year old man present to a rheumatologist
with complaints of joint pain involving the large * What is the m.common cause of bacterial UTI ?
joints of the legs . On questioning the patient A . Staphylococcus
indicates that exacerbations in the joint pain are B . Neisseria
frequently accompanied by diarrhea . Which of C . E coli
the following most likely diagnosis ? D . Streptococcus
A . Amebic colitis E . Pseudomonas
B . Chronic appendicitis
C . Diverticulosis * Which of the following is an appropriate initial
D . Pseudomembranous colitis treatment for pulmonary tuberculosis ?
E . Ulcerative colitis A . INH
B . INH and rifampin
* A 38 year old man has pulmonary tuberculosis , C . INH , rifampin and ethambutol
treatment with isoniazid , rifampin , ethambutol D . INH, rifampin , ethambutol and pyrazinamide
and pyrazinamide is started, which of the E . INH , rifampin , ethambutol and streptomycin
following should be added to the medication
regimen to prevent neurologic toxicity in this * The treatment of choice for recurrent transient
patient ? ischemic attack in patient on aspirin with new
A . Vitamin B9 onest atrial fibrillation is ?
B . Vitamin B3 A . Anticoagulation
C . Vitamin B12 B . Carotid endarterectomy
D . Vitamin B6 C . Clopidogrel
E . Vitamin C D . Corticosteroids
E . Carotid stent
* Major criteria of rheumatic fever are all except ?
A . Carditis * Chronic renal dialysis patient is brought to the
B .subcutaneous nodules emergency department incardiac arrest , the
C . Arthralgia most likely cause is ?
D . Erythema marginatum A . Pericardial effusion
E . Sydenham Chorea B . Hyperkalemia
C . Hypocalcemia
D . Malignant HTN
E . Post dialysis hypotension

* A 30 year old female is started on Carbimazole * All of the following are contraindications to
20 mg following a diagnosis of Graves disease . passing a nasogastric tube except ?
What is the best biochemical marker to assess A . Hx of esophageal varices
her response to treatment ? B . Basal skull fracture
A . Total T4 C . Suspected perforation of the esophagus
B . TSH D . Complete intestinal obstruction
C . Free T4 E . Presence of esophageal foreign body
D . ESR
E . Free T3 * All of the following are side effects of steroid
except ?
* A 75 years old woman has increasing shortness A . Elevated blood pressure
of breath on exertion . Findings on physical B . Acne
examination are unremarkable . X_Ray of the C . Skin bruises
chest show no abnormalities of the hear or lungs D . Hypoglycemia
Labs show HCT 28% , Hb 9 g/dl , MCV 70 . E . Cataract
Which of the following is most likely diagnosis ?
A . Acquired Hemolytic anemia * All of the following can cause acute renal
B . Chronic blood loss failure except ?
C . Folic acid deficiency A . Rhabdomyolysis
D . B _ thalassemia minor B . NSAIDs
E . Pernicious anemia C . Ethylene glycol
‫ اﺳﺎﻣﻪ اﻟﺨﺰاﻋﻠﻪ‬: ‫اﻟﺪﻛﺘﻮر‬ ‫اﺳﺌﻠــــــــﺔ اﻟﺒﺎﻃﻨﻴــــــﺔ‬
Page 7
D . Penicillin B . Push fast more than 100 and less than 120
E . Iron compressions per minute .
C . Minimize interruptions in compressions and
* Which of the following is the most common give shock as soon as possible .
cause of Duodenal ulcer ? D . Compressions to ventilation ratio 30 : 2 .
A . NSAIDs E . Target of CPR quality are ETco2 > 10 mmhg
B . H pylori and diastolic BP > 20 mmhg .
C . Alcohol abuse
D . Chronic corticosteroids * All of the following are common causes of
E . Zollinger Ellison syndrome chest pain except ?
A . Angina
* All of the following forms of vaccine except ? B . Pericarditis
A . Killed vaccine C . PE
B . Attenuated vaccine D . Tension pneumothorax
C . Toxoid E . Esophageal reflux
D . Serum
E . Subunit * All of the following are true matching about
drugs and side effects except ?
* A 19 year old female presents with sudden A . Sulfa drugs _ Steven Johnson syndrome .
sever right sided chest pain that develops shortly B . Aminoglycosides _ nephrotoxicity .
after she had been placing heavy boxes on C . Acetaminophen _ GI upset and bleeding .
shelves in her garage physical examination D . Opiates _ constipation .
reveals an a febrile female in mild respiratory E . Atenolol _ Bradycardia.
distress . Breath sounds are markedly decreased
on the right , and the right lung is hyperresonant * All of the following are causes of DM except ?
to percussion . Which of the following is most A . Thyroid hormone
likely present in this individual ? B . Thiazides
A . Pneumoconiosis C . Pancreatic cancer
B . Pneumocystis infection D . Addison disease
C . Bacterial pneumonia E . Hemochromatosis
D . Viral pneumonia
E . Pneumothorax * All of the following are differential diagnosis of
ST segment elevation except ?
A . Acute Pericarditis
B . Early repolarization
C . Hyperthermia
D . Prinzmetal angina
E . Left ventricular aneurysm
* Tertiary prevention consist of ?
A . Crisis preparation * All of the following are rapid or short acting
B . Elimination of specific disease prandial bolus insulin except ?
C . Early diagnosis A . Insulin glargine
D . Rehabilitation after disability B . Insulin regular
E . Environmental sanitation C . Insulin lispro
D . Insulin glulisine
* Which of the following statement about E . Insulin aspart
Polyvalent anti_snake venom serum is true ?
A . It is to be given subcutaneously * Adverse consequences of obesity include all of
B . It is an example of passive immunization the following except ?
C . It is usually obtained from human's donor's A . Coronary artery disease
D . Should be stored at 0 Sep Celsius B . Hypertension pulmonary
E . It gives life long immunity C . DM type one
D . Osteoarthritis
* All are true about CPR quality in advanced E . Colorectal cancer
cardiac life support except ?
A . Push hard less than 2 inches and allow * NT_PRO BNP elevation is caused by all of the
complete chest recoil . following except ?
A . MI
‫ اﺳﺎﻣﻪ اﻟﺨﺰاﻋﻠﻪ‬: ‫اﻟﺪﻛﺘﻮر‬ ‫اﺳﺌﻠــــــــﺔ اﻟﺒﺎﻃﻨﻴــــــﺔ‬
Page 8
B . CHF * All of the following are matching about
C . Atrial fibrillation hepatitis B serology except ?
D . PE A . Acute infection _ HBsAg , HBeAg , anti HBc
E . COPD exacerbation IgG are positive .
B . Chronic inactive infection _ HBsAg , HBeAg ,
* All are true about Hyperosmolar hyperglycemic anti HBc IgG are positive .
state except ? C . Chronic active infection _ HBsAg , anti HBe ,
A . Partial or relative insulin deficiency . anti HBc IgG are positive .
B . Characterized by hyperglycemia , D . Resolved infection _ anti HBs , anti HBe , anti
hyperosmolarity , dehydration and ketosis . HBc IgG are positive .
C . Occurs in DM type two . E . Immunized person _anti HBs positive .
D . Lower insulin requirement compared to DKA .
E . Overall mortality higher compared to DKA . * Auer rod white blood cell on blood film is
characteristic for ?
* All are causes of Acute bloody diarrhea except A . AML
A . Shigella B . CML
B . Campylobacter C . ALL
C . Bacillus cereus D . CLL
D . Yersinia E . Hodgkin lymphoma
E . Entamoeba histolytica
* Mode of transmission for all of the following by
* All of the following are indications for Enalapril food_water borne except ?
administration except ? A . MRSA
A . HTN B . Cholera
B . MI C . Salmonella
C . CHF D . HAV
D . DM E . HEV
E . Aortic stenosis
* Red currant jelly sputum is characteristic for ?
* All are risk factors for Barrett esophagus except A . Streptococcal pneumonia
A . Old age B . Staphylococcal pneumonia
B . Female C . Klebsiella pneumonia
C . Smokers D . Mycoplasma pneumonia
D . Obesity E . Legionella pneumonia
E . Hiatus hernia
* All of the following are causes of
Reticulocytosis except ?
A . SCA
B . Chronic blood loss
C . HUS
* All are true about Crohn disease except ? D . DIC
A . Usually affect terminal ileum . E . Malaria
B . Fever is very common . * All of the following are causes of cavitating
C . Palpable mass is rare compared to ulcerative lung nodule on CXR except ?
colitis . A . Wegener disease
D . Diarrhea less prevalent than ulcerative colitis B . Allergic alveolitis
E . Cobblestone mucosal appearance . C . Sarcoidosis
D . Tuberculosis
* All of the following are true about irritable E . Metastasis
bowel syndrome except ?
A . Onest of symptoms usually in young adults . * All of the following are true matching about the
B . Signs and symptoms relived with defecation , interpretation of urinary casts except ?
increased by stress . A . Fatty casts _ nephritic syndrome .
C . Bloody diarrhea . B . Pigmented epithelial granular cast _ ATN .
D . Inflammatory markers are negative . C . White blood cell casts _ interstitial nephritis .
E . 80% improve over time . D . Red blood cell casts _ glomerulonephritis .
E . Hyaline casts _ physiological during exercise
or fever
‫ اﺳﺎﻣﻪ اﻟﺨﺰاﻋﻠﻪ‬: ‫اﻟﺪﻛﺘﻮر‬ ‫اﺳﺌﻠــــــــﺔ اﻟﺒﺎﻃﻨﻴــــــﺔ‬
Page 9
A . Protect health and lifestyle
* All of the following are true matching about B . Prevention of disease Onest
RBC inclusions except ? C . Early detection of disease
A . Erythroblasts _ Hemolytic anemia . D . Treat and rehabilitate of disease to prevent
B . Heinz bodies _ IDA progression , and disability .
C . Howell jolly bodies _ post splenectomy E . Stop of smoking .
D . Basophilic stippling _ lead poisoning .
E . Sideroblasts _ hypothyroidism * All of the following are causes of
Hyperglycemia except ?
* Westermark sign , Hamptons hump , enlarged A .DM
right ventricle and right atrium and atelectasis on B .Pheochromocytoma
CXR is characteristic for ? C .Thyrotoxicosis
A . Pulmonary edema D . DI
B . Pulmonary embolism E . Glucagonoma
C . Sarcoidosis
D . COPD * All of the following are causes of hypoglycemia
E . Pulmonary fibrosis except ?
A .Glucagonoma
* All of the following are causes of Hyponatremia B .Postprandial
except ? C .Addison disease
A . Nephrotic syndrome D . Glibenclamide
B . Hyperthyroidism E. Liver cirrhosis
C . SIADH
D . Furosemide * All of the following are causes of Hyponatremia
E . CHF except ?
A . SIADH
* Signs and symptoms of third cranial nerve B . Furosemide
palsy include all of the C . DKA
following except ? D . DI
A . Absent of light reflexes E . Addison disease
B . Ptosis
C . Eye deviated downward * About SIADH , all are true except ?
D . Eye deviated outward A . Euvolemic state
E . Mydriasis B . High urine specific gravity
C . High urine sodium
* Paresthesia , sensory ataxia , myopathy , D . Hypernatremia
subacute combined degeneration of spinal cord , E . Low serum osmolality
optic neuropathy are characteristic of ?
A . Vitamin B12 deficiency * Most c. cause of hypothyroidism worldwide ?
B . Vitamin B6 deficiency A . Hashimoto thyroiditis
C . Vitamin B1 deficiency B . Gravis disease
D . Folate deficiency C . Iodine deficiency
E . Niacin deficiency D . Subacute thyroiditis
* All are causes of Hypokalemia except ? E . Post_thyroidectomy
A . Bartter syndrome * The best laboratory test for follow up patient
B . Angelman syndrome with Hashimoto thyroiditis is ?
C . Hypomagnesemia A . T3
D . Conn syndrome B . T4
E . Cushing syndrome C . TSH
D . TRH
* One is true about Petit mal seizures ? E . ESR
A . Muscle rigidity
B . Tongue biting * All are true about hyperthyroidism except ?
C . Arrest of activity for 5 _ 10 seconds . A .Increased appetite
D . Flaccid limbs B . Heart rate 50 BPM
E . Loss of muscle tone C . Hypomenorrhea
D . TSH is very low
* The goal of tertiary level of prevention is ? E . Atrial fibrillation on ECG
‫ اﺳﺎﻣﻪ اﻟﺨﺰاﻋﻠﻪ‬: ‫اﻟﺪﻛﺘﻮر‬ ‫اﺳﺌﻠــــــــﺔ اﻟﺒﺎﻃﻨﻴــــــﺔ‬
Page 10
E . Addison disease
* All are causes of hypercalcemia except ?
A .Thyrotoxicosis * All of the following are causes of Hyperkalemia
B .Multiple myeloma except ?
C . Lung cancer A . Nelson syndrome
D . Furosemide B . Rhabdomyolsis
E . Hyperparathyroidism C . Digoxin toxicity
D . Enalapril
* 55 years old male patient with with back pain , E . Conn syndrome
spine X_Ray shows fracture in body of lumbar
vertebrae . Lab findings : K 4.1 , Na 140 , Cl 101 , * One of the following statements is wrong ?
Ca 13 , Hb 8.5 , WBC 12.3 , platelets 150*10`9 , A . Patient with Pheochromocytoma must be
total protein 105 g/dl . Which of the following is treated by B_Blockers
the most likely diagnosis ? B . Pheochromocytoma is one of the causes of
A .Sarcoidosis resistant HTN .
B . Osteosarcoma C . Most common cause of secondary HTN is
C . Multiple myeloma renal disease .
D . Age related changes D . Conn syndrome is primary
E . Paget disease of the bone hyperaldosteronism .
E . Most common cause of Thyrotoxicosis is
* The main pathophysiology of Myasthenia Graves disease .
gravis is ?
A . Anti calcium channel bodies * Amlodipine is ?
B . Low level of acetylcholine A . Aldosterone antagonist .
C . Hyposensitivity to acetylcholine B . B blockers .
D . Acetylcholine receptor antibodies C . Muscle relaxants .
E . Neuromuscular degeneration changes D .Dihydropyridine CCB
E . Non Dihydropyridine CCB .
* All are causes of Polyuria except ?
A . DM * 50 years old male patient is presented with
B . DI fasting blood sugar 139 mg/dl , which of the
C . Renal failure following is most appropriate next step ?
D . Hypercalcemia A . GAD study .
E . SIADH B . Postprandial glucose tolerance test .
C . Start treatment right now as DM .
* All are true about resistant HTN except ? D . Another reading of fasting blood sugar
A . It is diuretics related condition E . Diet therapy .
B . Pheochromocytoma is underlying cause .
C . To put this diagnosis , patient must be on 3 * All are true about DKA except ?
drugs , one of them Thiazide . A . PH of the serum < 7.35
D . To put this diagnosis , patient must be take B . Hyponatremia
the optimal dose for HTN drugs . C . Ketones bodies in serum
E . Diet problems is underlying cause . D . Normal anion gap
E . Complicated by brain edema

* All are causes of Thrombocytosis except ? * The major deference between primary and
A . IDA secondary adrenal insufficiency is ?
B . Polycythemia rubra vera A . Na and k in serum
C . Hypersplenism B . Blood pressure
D . After Splenectomy C . Weight status
E . Collagen disorders D . Glucose in serum
E . Skin pigmentation
* All of the following are causes of Hypokalemia
except ? * Criteria of disease activity for rheumatoid
A . Cushing syndrome arthritis include all of the following except ?
B . Conn syndrome A . Rheumatoid factor
C .Thiazide B . ESR
D .Pyloric stenosis C . CRP
‫ اﺳﺎﻣﻪ اﻟﺨﺰاﻋﻠﻪ‬: ‫اﻟﺪﻛﺘﻮر‬ ‫اﺳﺌﻠــــــــﺔ اﻟﺒﺎﻃﻨﻴــــــﺔ‬
Page 11
D . Thrombocytosis C . Varicose veins of esophagus
E . Anti_CCP D . Esophagitis
E . Gastric angiodysplasia
* All of the following about TB are true except ?
A . This disease is preventable and treatable . * Best tool for diagnosis of infective endocarditis
B . This disease is systemic A . ECG
C . This disease mainly affect the Lower lobes of B . CT angiography
the lung C . Mediastinoscopy
D . BCG vaccine is live attenuated vaccine for TB D . MRI
E . Most patients TB infections are due to E . Two dimensional echo
secondary infection
* All are causes of chronic liver disease except ?
* Obesity associated with all of the following A . Hepatitis B
except ? B . Hemochromatosis
A . Psychological disorders C . Alpha one antitrypsin deficiency
B . GIT disturbances D . Hepatitis A
C . Colorectal cancer E . Cystic fibrosis
D . Osteoporosis
E . Osteoarthritis * All are complications of cystic fibrosis except ?
A . Pancreatic insufficiency
* All are causes of Leucocytosis except ? B . Hypoglycemia
A . Dermatomyositis C . Recurrent lung disease
B . Polymyositis D . Recurrent sinusitis
C . Fibromyalgia E . Anal prolapse
D . Septic arthritis
E . Polyarteritis nodosa * All are causes of Bradycardia except ?
A . Metoprolol
* All are causes of microcytosis except ? B . Digoxin
A . Pernicious anemia C . Thyroxine
B . Sideroblastic anemia D . Athletic person
C . B thalassemia E . Neostagmine
D . Lead toxicity
E . Anemia of chronic disease * Risk of transmission of HIV to medical person's
from needle stick is ?
* 22 years old patient with celiac disease , A . 0.3%
admitted with chief complain of diarrhea for 2 B . 0.03%
weeks , the most likely cause is ? C . 3%
A . GIT infection D . 30%
B . IBS E . 10%
C . IBD
D . Carcinoid syndrome * About HIV , all are true except ?
E . No adherent to gluten free diet A . AIDS is the last stage of HIV infection .
B. HIV infection is not preventable by vaccination
C . Heterosexual type of transmission is the most
common
* 25 years old male patient present with multiple D . It is transmitted by blood transfusion .
psychological problems , which of the following * Paracetamol antidote is ?
disturbances is the most likely to see in this A . Protamine sulfate
patient ? B . N_acetylcysteine
A . Bloody diarrhea C . Flumazenil
B . GIT ulcers D . Bicarbonate
C . Occult blood in stool E . Vitamin K
D . Alternating diarrhea and constipation
E . IDA * About pulmonary embolism , all are true except
A . Past history of DVT support the diagnosis .
* Most common cause of upper GI bleeding is ? B . Heparin is used for treatment
A . Mallory Weiss syndrome C . Chest X_Ray usually normal without
B . PUD significant changes
‫ اﺳﺎﻣﻪ اﻟﺨﺰاﻋﻠﻪ‬: ‫اﻟﺪﻛﺘﻮر‬ ‫اﺳﺌﻠــــــــﺔ اﻟﺒﺎﻃﻨﻴــــــﺔ‬
Page 12
D . It is a block in one or more of the pulmonary C . Edema
veins D . Hypercholesterolemia
E . Classical ECG finding is S1Q3T3 E . Hypoalbuminemia

* Most important risk factors for stroke * The most important finding in
development is ? Glomerulonephritis is ?
A .High blood pressure A . WBCs casts in urine
B . Low blood pressure B . Hyaline casts in urine
C . Aneurysm rupture C . RBCs casts in urine
D . Trauma D . Proteinuria
E . Vasculitis E . Bacteriuria

* All are causes of increased CVP except ? * Best drug that is used for prophylaxis and
A . Fluid overload treatment of diabetic nephropathy is ?
B . Right sided heart failure A . ARBs
C . Pulmonary HTN B . B blockers
D . Anaphylactic shock C . CCB
E . Pulmonary stenosis D . Furosemide
E . Nitrofurantoin
* All are used in treatment of Anaphylactic shock
except ? * Best way of the following to prevent diabetic
A . Adrenaline retinopathy is ?
B . IV fluid A . Optimal dose of insulin
C . 100 % oxygen B . Optimal dose of metformin
D . Hydrocortisone C . Annual visit to physician
E . Propranolol D . ACE_i administration
E . Good control of sugar
* The major deference between vitamin B12 and
folate deficiency anemia is ? * The most early and best laboratory finding for
A . MCV value diagnosis of diabetic nephropathy is ?
B . HCT value A . Urine analysis positive for Bacteria
C . Blood film findings B . Kidney biopsy
D . Foot Paresthesia C . RBCs casts in urine
E . MCHC value D . 24 hour protein /creatinine ratio in the urine
E . Kidney ultrasound
* All are true about Mycoplasma pneumonia
except ? * All of the following are causes of increasing
A . Patient is looking well . Triglycerides in serum except ?
B . Associated with Hemolytic anemia . A . Nephrotic syndrome
C . Common in younger age group . B . Ibuprofen
D . Good response for Macrolides antibiotics . C . DM
E . Associated with septic shock D . Alcohol
E . Lipoprotein lipase deficiency

* Highly severe pneumonia depends on all of the * All of the following represents targets for
following criteria except ? systemic HTN except ?
A . BUN > 7 mmol/l A . Retinopathy
B . Confusion B . Nephropathy
C . Age 20_30 years old C . Heart failure
D . R.R > 30/min D . Subdural hemorrhage
E . Systolic blood pressure < 90 mmHg E . Peripheral vascular disease

* All of the following are true about Nephrotic * All of the following are causes of exudative
syndrome except ? pleural effusion except ?
A . HTN A . PE
B . Proteinuria > 3.5 gram/day B . Hypoalbuminemia
‫ اﺳﺎﻣﻪ اﻟﺨﺰاﻋﻠﻪ‬: ‫اﻟﺪﻛﺘﻮر‬ ‫اﺳﺌﻠــــــــﺔ اﻟﺒﺎﻃﻨﻴــــــﺔ‬
Page 13
C . Lymphoma B . Myelofibrosis
D . Mesothelioma C . Malaria
E . SLE D . Leishmaniasis
E . Tropical Splenomegaly
* All of the following are true about COPD ,
except ? * All of the following are risk factors for hepatic
A . FEV1< 80% of predicted encephalopathy except ?
B . Complicated by right heart failure A . GIT bleeding
C . High F:O2 is needed B . Benzodiazepine
D . Stop of smoking is beneficial in stop C . Rifampicine
progression of the disease D . Diuretics
E . In ABG PaO2 is low , PaCO2 is high . E . Neomycin

* All of the following are increases risk of DVT * All are true about acute renal failure except ?
except ? A . Pre_renal causes are the m. common causes
A . Malignancy B . ATN is the most common renal cause
B . Estrogen therapy C . It is rarely caused by enlarged prostate
C . Old age D . Small kidneys and increased echogenicity on
D . Protein C excess ultrasound
E . Protein S deficiency E . Usually no history of Co_morbidity

* In Hyperventilation syndrome , one is true ? * Long term steroid therapy can cause all of the
A . Increasing PH of the serum following except ?
B . Caused by severe attack of bronchial asthma A . Hypernatremia
C . Characterized by hypercapnia B . Hyperkalemia
D . Caused by poliomyelitis C . GIT ulcers
E . No relation to acute respiratory distress D . Suprarenal gland failure
syndrome E . Osteoporosis

* 65 years old male patient , present to * About DKA , all are true except ?
emergency room with chief complain of right A . Hyponatremia
sided weakness and speech inability , one B . Leucocytosis
physical examination, right side weakness 1/5 , C . Serum PH decreased
speech exam , patient obey command but can't D . PaCO2 decreased
speak any words , this type of dysphasias is E . Normal anion gap
most likely ?
A . Receptive aphasia * In severe bronchial asthma , all of the following
B . Conduction aphasia are true except ?
C . Expressive aphasia A . Cyanosis
D . Nominal aphasia B . Confusion
E . Dysarthria C . Hyperresonant chest
D . Exhaustion
All are components of Horner syndrome except ? E . Bradycardia
A . Proptosis
B . Myosis
C . Anhidrosis
D . Enophthalmos
* All of the following are decrease potassium in
Hyperkalemic patient except ? * The target HbA1c in diabetic patient is ?
A . Insulin and glucose A . < 6%
B . Ventolin nebulizer B . < 7%
C . Furosemide C . < 8%
D . Calcium gluconate D . < 9%
E . Kayexalate resin E . < 5%

* All of the following are causes of huge massive * All are causes of Hypokalemia except ?
Splenomegaly except ? A . Spironolactone
A . Hemolytic anemia B . Furosemide
‫ اﺳﺎﻣﻪ اﻟﺨﺰاﻋﻠﻪ‬: ‫اﻟﺪﻛﺘﻮر‬ ‫اﺳﺌﻠــــــــﺔ اﻟﺒﺎﻃﻨﻴــــــﺔ‬
Page 14
C . Thiazide
D . Conn syndrome * Drug of choice to treat patient with HTN and
E . Cushing syndrome bronchial asthma is ?
ACE_i
* All are true about inflammatory diarrhea except B blockers
A .Stool PH is low with alkalosis . CCB
B .Stool usually watery . Statin
C.Consider IBD, c.Difficile infection .
D .Stool volume less than 500 ml . * All are true about Ankylosing spondylitis except
E .Continue during fasting, associated with A . More common in male .
anemia , urgency . B . Aortitis and AV block are manifestation .
C . Unilateral uveitis with painful red eye ,
* All are true about metformin except ? Photophobia .
Decreased insulin resistance , so it can be given D . Obstructive lung disease is manifestation .
in PCOS . E . Associated with HLA_B27 gen .
It teratogenic , so it can't be given in gestational
diabetes . * All of the following are characteristic of IDA
Most serious side effects is lactic acidosis , so it except ?
can't be given in renal failure . Anisocytosis
It inhibits gluconeogenesis in the liver . Spherocytosis
No effect on pancreas . Poikilocytosis
Hypochromia
* Which of the following causes of Potts disease Microcytosis
Primary TB
Skeletal TB * Most common cause of travelers diarrhea is ?
TB meningitis E coli
Miliary TB Cholera
GIT TB Shigella
Brucella
* Patient presents with COPD , Hepatomegaly, Staphylococcus aureus
peripheral edema , finger clubbing . ECG : peaked
P wave , most likely diagnosis ? * One of the following seen in esophageal
PE perforation ?
Cor pulmonale Courvoisier sign
LVF Grey sign
LAH Cullen sign
Sleep apnea Boerhaave syndrome
Achalasia
* All of the following are true about Sarcoidosis
except ? * All are signs and symptoms of hypothyroidism
More in black race . except ?
May cause complete heart block . Carpal tunnel syndrome
Involve mostly left hilar lymph nodes . Hyponatremia
Lupus pernio and Erythema nodosum are seen . Pericardial effusion
Hepatosplenomegaly is common . Hypolipidemia
Normochromic Normocytic anemia

* The criteria for diagnosis chronic bronchitis is ?


A . Cough productive for 2 months of two * Most common cause of gynecomastia is ?
successive years . Liver failure
B . Cough productive for 2 months of three Estrogen secreting tumor
successive years . HCG secreting tumor
C . Cough productive for 3 months of two Idiopathic
successive years . Puberty
D . Cough productive for 3 months of three
successive years . * All are characteristic for MEN1 except ?
‫ اﺳﺎﻣﻪ اﻟﺨﺰاﻋﻠﻪ‬: ‫اﻟﺪﻛﺘﻮر‬ ‫اﺳﺌﻠــــــــﺔ اﻟﺒﺎﻃﻨﻴــــــﺔ‬
Page 15
Hypercalcemia * The most appropriate drug to use in case of
Hypoglycemia thyroid storm is ?
PUD Methimazole
Pheochromocytoma Dexamethasone
Glucagonoma Propylthiouracil
Supportive care
* Most effective drug increasing HDL is ? Iodine
Statins
Fibrate * All of the following are true about DKA except ?
Bile acid resins Hypovolemia
Niacin Kassmaul breathing
Ezetimibe Coma
Hyperglycemia
* Red man syndrome is a side effect of ? Low anion gap metabolic acidosis
Vancomycin
Amoxicillin * All are causes of Hypocalcemia except ?
Metronidazole Acute pancreatitis
Levothyroxine Hypermagnesemia
Levofloxacine Pseudo_hypoparathyroidism
Hyperphosphatemia
* Most common site of tuberculosis in Jordan Chronic renal failure
is ?
Skeletal * First step in diagnosis Acromegaly is one of the
Meningitis following test in serum ?
Lymphatic IGF_1
Pulmonary IGF_2
Miliary G.H
G.H.R.H
* Most common antibiotics to cause anaphylaxis Somatostatin
Aminoglycosides
B_lactam * Most common manifestation of Brucellosis ?
Cephalosporin Bradycardia
Glycopeptide Abdominal pain
Macrolides Fever
Dry cough
* An injury of optic chiasm by pituitary adenoma Skin rash
will cause ?
Monocular visual loss * Most common cause of Cor pulmonale is ?
Bitemporal hemianopia Pulmonary embolism
Homonymous hemianopia Obstructive sleep apnea
Inferior Homonymous quadratanopia COPD
Bronchial asthma
* Which of the following muscles is supplied by Tuberculosis
Abducens nerve ?
Superior rectus * All of the following are symptoms of bronchial
Medial rectus asthma except ?
Lateral rectus Dry cough
Inferior rectus Chest tightness
Superior oblique Wheezing
Stridor
Shortness of breath
* Most specific test for diagnosis
Pheochromocytoma is ? * All are true about Sarcoidosis except ?
CT scan A . Restrictive lung disease .
Dexamethasone test B . Hypocalcemia .
24 hour urine metanephrines C . Non caseating Granuloma on biopsy .
Plasma free metanephrines D . Hepatosplenomegaly .

‫ اﺳﺎﻣﻪ اﻟﺨﺰاﻋﻠﻪ‬: ‫اﻟﺪﻛﺘﻮر‬ ‫اﺳﺌﻠــــــــﺔ اﻟﺒﺎﻃﻨﻴــــــﺔ‬


Page 16
E . Painless skin sores on face which called lupus Aortoangiography
pernio .
* All of the following are side effects of Thiazide
* All of the following are signs of pulmonary diuretic except ?
hypertension except ? Hypokalemia
A . Right ventricular hypertrophy . Hypocalcemia
B . Prominent A wave on JVP . Hypomagnesemia
C . Loud S1 Hyponatremia
D . Hepatosplenomegaly and Ascites . Hyperglycemia
E . Systolic pulmonary pressure > 30 mmHg and
mean pulmonary pressure > 20 * Drug of choice to treat Raynauds phenomenon
mmHg . ACE_i
B blockers
* LMWH works on ? CCB
Factor 2 Loop diuretics
Factor 7 ARBs
Factor 8
Factor 10 * All of the following are parts of atrial fibrillation
Factor 12 management except ?
Class 1A Quinidine
* All are causes of lower limbs edema except ? Class 1B lidocaine
Cellulitis Class 1C propafenone
Ruptured Baker cyst Class 3 Amiodarone
Lymphedema DC shock
Left heart failure
DVT * All of the following must be treated with DC
shock except ?
* Which of the following is contraindicated in Unstable atrial fibrillation
case of variant prenzmetal Pulsless ventricular tachycardia
angina ? Ventricular fibrillation
ACE_i Unstable fibrillation
ARBs PVC
B blockers
CCB * Carotid thrills , arterial pulsation in retinal artery
GTN ( Becker sign ) , head nodding , water hummer
pulse , bisfirenes pulse , pistol shot pulse are
* Most common cause of secondary HTN is ? seen ?
Chronic kidney disease Mitral stenosis
Cushing syndrome Mitral regurgitation
Hyperthyroidism Aortic stenosis
Obstructive sleep apnea Aortic regurgitation
Conn syndrome Pulmonary stenosis

* Most common cause of infective endocarditis ? * Most common valve disease causing
Streptococcus group A pulmonary HTN ?
Streptococcus group B Mitral stenosis
Streptococcus viridans Mitral regurgitation
Staphylococcus viridans Aortic stenosis
Staphylococcus group A Aortic regurgitation
Mitral valve prolapse

* Most common way to diagnose aortic stenosis


CT angiography * All are risk factors for developing infective
MRA endocarditis except ?
Transesophageal echocardiography AIDS
‫ اﺳﺎﻣﻪ اﻟﺨﺰاﻋﻠﻪ‬: ‫اﻟﺪﻛﺘﻮر‬ ‫اﺳﺌﻠــــــــﺔ اﻟﺒﺎﻃﻨﻴــــــﺔ‬
Page 17
VSD A . Most common manifestation is painful oral
HOCM apththous ulcer .
ASD B . Erythema toxicoum
Prosthetic valve C . DVT and PE
D . HLA_B51 gen is positive
* Most sensitive test to diagnose SLE is ? E . Mostly seen in Turkish people age 20_30
RF years old .
ESR
Ds_DNA * Philadelphia chromosome is mostly seen in
ANA which of the following ?
Anti_smith A . Acute myeloblastic leukemia .
B . Acute lymphoblastic leukemia .
* Which of the following can be used for C . Chronic myeloblastic leukemia .
diagnosis of Wegener Granulomatosis ? D . Hodgkin lymphoma
C_ANCA antibodies E . Non Hodgkin lymphoma
P_ANCA antibodies
Anti_RNP * Carbamezipine can be used in all of these
ANA antibodies cases except ?
Anti histone antibodies Partial seizure simple
Generalised tonic clonic seizure
* Most common SLE manifestation is ? Myoclonic or absence seizure
Skin Trigeminal neuralgia
Joints Complex partial seizure
Neurology
Renal * An injury to the optic tract will cause ?
Blood A .Monocular visual loss
B .Bitemporal Hemianopia
* All are true about reactive arthritis Reiters C .Homonymous hemianopia
syndrome except ? D .Inferior Homonymous quadratanopia
Keratoconjunctivitis sicca and uveitis . E.Superior Homonymous quadratanopia
Balanitis and oral ulcer .
HLA_B27 is positive and RF is positive . * Which of the following can be used to
GIT infections are the only cause . differentiate between Myasthenia gravis and
Kerato_derma blenorrhagica Lambert Eaton Myasthenic syndrome ?
X_Ray
* Most specific antibodies in SLE ? CT scan
Anti ss _ A MRI
Anti ss _ B ECG
Anti _ RNP EMG
Anti Ds_DNA
ANA * First line drug to be used in management a
patient with migraine headache is ?
* Which of the following large vessel pathology ? Ergotamine
A . HSP Sumatriptan
B . Takayasu arteritis Propranolol
C . Wegener Granulomatosis Verapamil
D . Churg Strauss syndrome Lithium
E . Kawasaki disease
* Most common cause of epilepsy is ?
* Most specific test for diffuse systemic Stress
sclerosis is ? Congenital
SCL_70 Trauma
Anti_centromere antibodies Idiopathic
ANA Hypoxia
Anti RNP
Anti histone antibodies
* All are true about Behcets disease except ?

‫ اﺳﺎﻣﻪ اﻟﺨﺰاﻋﻠﻪ‬: ‫اﻟﺪﻛﺘﻮر‬ ‫اﺳﺌﻠــــــــﺔ اﻟﺒﺎﻃﻨﻴــــــﺔ‬


Page 18
* The treatment of choice of Juvenile Myoclonic * All of the following are true about Brown
epilepsy ( Janz syndrome ) is ? Sequard syndrome except ?
Carbamezipine A . Upper motor neuron weakness on the same
Sodium Valproate side as the lesion .
Phenytoin B . Loss of position and vibration ipsilateral to
Corticosteroids the lesion .
Clonazepam C . Touch in intact .
D . Loss of pain and temperature ipsilateral to the
* All are true about migraine headache except ? lesion .
A . More common in teenage men .
B . Stress , exercise , cheese , chocolate , * Uhthoff phenomenon is seen in ?
menarche are triggering factors . Parkinson disease
C . Associated with aura . Meningitis
D . Unilateral associated with vomiting and Multiple sclerosis
Photophobia . Guillain barre syndrome
E . Maximum last for 3 days . Duchenne muscular dystrophy

* The treatment of choice for infantile spasms * Which of the following disease are associated
( west syndrome ) ( Hypsarrhythmia on ECG ) is ? with both degeneration in
Carbamezipine upper motor neuron and lower motor neuron ?
Sodium Valproate Parkinson disease
Phenytoin Multiple sclerosis
Corticosteroids Alzheimer's disease
Clonazepam Amyotrophic lateral sclerosis
Duchenne muscular dystrophy
* Best way to differentiate between cerebral
infraction and hemorrhage is ? * ALL of the following are associated with SLE
CT scan except :
X Ray A) Treatment include methotrexate
MRI B ) Cardiovascular disease is the most common
Angiogram cause of death .
MRA C) erosive arthritis
D) Libman sacks endocarditis
* All are true about Parkinson disease except ? E) Malar butterfly rash
A . Degeneration of extrapyramidial pathway .
B . Decreased dopamine level in brain . * All of the following are causes hyperurecemia
C . Triad Akinesia , tremor resting , rigidity . except :
D . Mean age of onest 60 years old . A) Hypothyroidism
E . Pathology is found in substantia gelatinosa . B) Lesh Nyhan syndrome
C) Thiazide diuretic
* Most common cause of meningitis in epidemic D) Hyperthyroidism
area is ? E) INH
Neisseria meningitis
Streptococcus pneumonia * Miosis associated with all of the following
Tuberculosis except :
Hib A) Organophosphates poisoning
Staphylococcus aureus B) Morphine
C) Oculomotor nerve palsy
* Drug of choice for prevention of close contacts D) Horner syndrome
with patient with E) Cluster headache
meningococcal meningitis is ?
Corticosteroids * All of the following are associated with diarrhea
Ceftriaxone except :
Rifampicine A) Hyperparathyroidism
Benzylpenicillin B) Celiac disease
Vancomycin C) Inflammatory bowel disease
D) Gastrinoma
E) Cryptosporidium
‫ اﺳﺎﻣﻪ اﻟﺨﺰاﻋﻠﻪ‬: ‫اﻟﺪﻛﺘﻮر‬ ‫اﺳﺌﻠــــــــﺔ اﻟﺒﺎﻃﻨﻴــــــﺔ‬
Page 19
D) Simvastatin
* All are true about multiple sclerosis except : E) Ramiprile
A) Occur in elderly * Which of the following valvular heart disease
B) Remission and relapse increase risk of angina :
C) Unilateral optic neuritis A) Aortic regurgitation
D) Antibodies to oligodendrocyte glycoprotein B) Mitral regurgitation
E) Lhermitts sign and uhthoffs sign are positive C) Aortic stenosis
D) Mitral stenosis
* Most common cause of early death E) Pulmonary stenosis
postoperative MI :
A) Ventricular extrasystole * All of the following are causes of chest pain
B) Myocardial rupture except :
C) Myocardial aneurysm A) Dissecting aortic aneurysm
D) Ventricular arrhythmia B) PE
E) Cardiac tamponade C) Pericarditis
D) Cardiomyopathy
* First line in management of ventricular E) Tietzes syndrome
fibrillation :
A) Adrenaline * All of the following associated with aortic
B) Atropine stenosis except :
C) Amiodarone A) Sudden death
D) DC shock B) Wide pulse pressure
E) Lidocaine C) Slow rising pulse
D) P mitrale and LAD on ECG
* All of the following feco_oral transmission E) Syncope
except :
A) HEV * All of the following are causes of huge
B) HDV Splenomegaly except :
C) Poliomyelitis A) CML
D) Gardiasis B) Kala_azar
E) Cryptosporidium C) Malaria
D) Sarcoidosis
* All of the following causes Polyuria except : E) Infectious mononucleosis
A) DI
B) Renal failure * All of the following are causes of Extrasystole
C) Insulin deficiency (PVCs) except :
D) SIADH A) Digoxin toxicity
E) Hypercalcemia B) MI
C) Lidocaine
* All are side effects of digitalis toxicity except : D) Heart failure
A) Nausea and vomiting E) HTN
B) Yellow vision
C) Short PR interval * All of the following are associated with
D) Gynecomastia Parkinsonism except :
E) Anorexia A) Bradykinesia
B) Mask like face
* All of the following are indication for liver C) Sensory defect
biopsy except : D) Shuffling gait
A) Hepatic tumor E) Lewy bodies
B) Wilson's disease
C) Prolonged PT * All are seen in DM type 1 except :
D) Fever unknown origin A) HLA DR3 and HLA DR4 association
E) Autoimmune hepatitis B) Obesity
C) Ketoacidosis
* Which of the following is CCB : D) Young age
A) Furosemide E) Autoimmune B cell damage
B) Diltiazem
C) Digoxin
‫ اﺳﺎﻣﻪ اﻟﺨﺰاﻋﻠﻪ‬: ‫اﻟﺪﻛﺘﻮر‬ ‫اﺳﺌﻠــــــــﺔ اﻟﺒﺎﻃﻨﻴــــــﺔ‬
Page 20
* Most common symptom of Bronchogenic
carcinoma :
A) Chest pain * All are causes of chorea except :
B) Cough A) PRV
C) Hemoptysis B) Rheumatic fever
D) Dyspnea C) Carbamezipine
E) Wheeze and cough D) Huntington disease
E) Wilson disease
* Rate of transmission of HBV by contaminated
needles is : * All of the following are causes of hemoptysis
A) 0.3% except :
B) 0.03 % A) Mitral stenosis
C) 0.003 % B) TB
D) 3% C) PE
E) 30% D) Lung cancer
E) Bronchial asthma
* Green house effect of increased temperature by
increase production of : * All of the following are viral disease except :
A) CO2 A) Rabies
B) O2 B) Poliomyelitis
C) CO C) Yellow fever
D) N2o D) Lyme disease
E) NO E) Dengue fever

* All of the following are complications of obesity * All are true about Brucellosis except :
except : A) Neutrophil Leucocytosis
A) Hypothyroidism B) Fever of unknown origin
B) DM type 2 C) Brucella melitensis is the most virulent
C) Polycythemia D) Weight loss
D) Obstructive sleep apnea E) Hepatosplenomegaly
E) Hyperlipidemia
* All are true about Nephritic syndrome except :
* One of the following associated with Addison A) Oliguria
disease : B) Diastolic HTN
A) Hypokalemia C) Lipiduria
B) Hyponatremia D) Uremia
C) Hypoglycemia E) Oedema
D) HTN
E) Weight gain * All are investigation for Thrombophilia except :
A) Anticardio lipin antibodies
* Gold standard for diagnosis of erosive B) Factor 5 leiden mutation
gastritis : C) Protein S level
A) Gastric PH_metry D) Cholesterol level
B) Gastric biopsy E) D_dimer
C) Gastroesophageal endoscopy
D) History * All are causes of ejection sys murmur except :
E) Physical exam A) pregnancy
B) Aortic stenosis
* Most common cause of septic arthritis : C) Pulmonary stenosis
A) Intra_Articular injection D) HOCM
B) Trauma E) Mitral stenosis
C) Staphylococcus aureus
D) Osteoarthritis * The physical sign of chest exam , decreased
E) Cushing syndrome chest expansion , increased percussion note
( Resonant ) , decreased breath sound in left side
of the lung are mostly :
‫ اﺳﺎﻣﻪ اﻟﺨﺰاﻋﻠﻪ‬: ‫اﻟﺪﻛﺘﻮر‬ ‫اﺳﺌﻠــــــــﺔ اﻟﺒﺎﻃﻨﻴــــــﺔ‬
Page 21
A) Left side pleural effusion A) Cefuroxime and metronidazole
B) Left side pneumothorax B) Amoxicillin and gentamycin
C) Left side lung fibrosis C) Vancomycin and ticaracillin
D) Left side consolidation D) Tetracycline and vancomycin
E) Left side collapse E) Gentamycin and metronidazole
* Ptosis , large pupils ,eye down and out are * All are causes of insulin resistance , DM type 2
characteristic for palsy of : except :
A) 2 C.N A) Wilson's disease
B) 3 C.N B) Pregnancy
C) 4 C.N C) Cushing disease
D) 5 C.N D) Metabolic syndrome
E) 6 C.N E) Acromegaly

* Chvostic sign is seen in one of the following : * Short 4th and 5th metacarpals bone is
A) Hypokalemia characteristic for :
B) Hyperkalemia A) Primary Hyperparathyroidism
C) Hypocalcemia B) Secondary Hyperparathyroidism
D) Hypercalcemia C) Tertiary Hyperparathyroidism
E) Hypermagnesemia D) Pseudohypoparathyroidism
E) Pseudopseudo_Hyperparathyroidism
* All are causes of finger clubbing except :
A) Empyema * All are causes of bloody diarrhea except :
B) Mesothelioma A) Cambylobacter
C) Celiac disease B) Shigella
D) Myocarditis C) Crohn disease
E) Graves disease D) E coli
E) IBS
* All are causes of Postural tremor except :
A) Parkinsonism * All are true about ATN except :
B) Hyperthyroidism A) Urine osmolarity > 500 mosm / L
C) Alcoholism B) Urine Na > 40 mmol / L
D) Anxiety C) Urine urea / plasma urea < 3:1
E) Pheochromocytoma D) Urine creatinine / plasma creatinine < 20:1
E) Fractional Na excretion > 2%
* All are true ECG sign for Hyperkalemia except
A) Tall tented T wave * All are causes of normocytic anemia except :
B) Wide QRS A) Chronic blood loss
C) Long QT interval B) Pregnancy
D) Absent P wave C) Hypothyroidism
E) Sine wave D) Alcoholism
E) Aplastic anemia
* All are sign and symptoms of digoxin toxicity
except : * All are true about IDA except :
A) Nausea and vomiting A) Low serum iron
B) Confusion B) Low TIBC
C) Decreased appetite C) Low serum ferritin
D) Hypokalemia D) Low hemosedrine
E) Edema E) High Transferrin

* All are true about complication of MI except : * All are causes of prolonged PT except :
A) Ventricular tachycardia A) DIC
B) Mitral stenosis B) Liver disease
C) VSD C) Warfarin
D) Dressler syndrome D) VWD
E) Heart block E) Factor 10 deficiency

* The best combination antibiotic for treatment * All are true matching about antibiotics and side
of aspiration pneumonia is : effect , except :
‫ اﺳﺎﻣﻪ اﻟﺨﺰاﻋﻠﻪ‬: ‫اﻟﺪﻛﺘﻮر‬ ‫اﺳﺌﻠــــــــﺔ اﻟﺒﺎﻃﻨﻴــــــﺔ‬
Page 22
A) Doxycycline _ photosensitivity
B) Vancomycin _ Red man syndrome
C) Clindamycin _constipation
D) Chloramphenicol _ Aplastic anemia
E) Metronidazole _-Disulfiram reaction
* All are sign and symptoms of autonomic * The first step in management of patient with
neuropathy except : Hyperkalemia with tall tented T
A) Urine retention wave :
B) Nocturnal diarrhea A) Calcium Resonium
C) erectile dysfunction B) Dialysis
D) HTN C) Calcium gluconate
E) Horner syndrome D) Insulin and glucose
E) Nebulized Salbutamol
* All of the following are causes of
polyneuropathy except : * All are cardiovascular complication of
A) Guillain barre syndrome acromegaly except :
B) DM A) HTN
C) Lyme disease B) HOCM
D) Charcot Marie tooth syndrome C) High output heart failure
E) Carpal tunnel syndrome D) Right ventricular hypertrophy
E) Coronary artery disease
* All are true about spondyloarthropathy except
A) Asymmetrical oligo or monoarthritis * All are true about mild acute attack of bronchial
B) HLA B27 association asthma except :
C) Extra articular manifestation A) Audible wheeze
D) Rheumatoid factor positive B) Hyperinflated chest
E) Affect spine and sacroiliac joints C) Hyperresonant percussion
D) Respiratory acidosis
* All of the following are causes of Nodular E) Cough
shadowing on chest X_Ray except :
A) Lung hamartoma * All are causes of type one respiratory failure
B) Hydatid cyst except :
C) Sarcoidosis A) Pneumonia
D) Caplan syndrome B) Pulmonary edema
E) Atelectasis C) Obstructive sleep apnea
D) Fibrosing alveolitis
* All are indication for D/C cardioversion except : E) Emphysema
A) Ventricular tachycardia
B) Ventricular fibrillation * All are causes of diffuse goitre except :
C) Atrial fibrillation A) Graves disease
D) Asystole B) Hashimoto thyroiditis
E) SVT C) Plummer disease
D) De Quervain thyroiditis
* All are true about life threatening attack E) Subacute thyroiditis
bronchial asthma except :
A) FEV1 < 50% of predicted * All of the following are causes of constipation
B) Cyanosis except :
C) Hypotension and bradycardia A) Hyperkalemia
D) Normal or high PaCO2 B) Hypercalcemia
E) PH < 7.30 C) Hypothyroidism
D) Iron supplement
* The first step of management of patient with E) Systemic sclerosis
DKA is :
A) Insulin * All are therapeutic indication for colonoscopy
B) K replacement except :
C) Fluid replacement normal saline A) FAP
D) Fluid replacement ringer lactate B) Hyperplastic polyps
E) Bicarbonate C) Angiodysplasia
‫ اﺳﺎﻣﻪ اﻟﺨﺰاﻋﻠﻪ‬: ‫اﻟﺪﻛﺘﻮر‬ ‫اﺳﺌﻠــــــــﺔ اﻟﺒﺎﻃﻨﻴــــــﺔ‬
Page 23
D) volvulus osmolarity in mosm/L H2o ?
E) Colonic pseudo_obstruction A . 24 , 314 .
B . 23 , 312 .
C . 32 , 321 .
D . 25 , 315 .
* The best screening test for hereditary E . 22 , 311 .
Hemochromatosis : * 50 years old male patient , with medical history
A) Serum iron of DM , free surgical history , social history of
B) Serum ferritin infertility , abdominal ultrasound showed cirrhotic
C) Serum Transferrin liver changes , most likely diagnosis ?
D) Serum Transferrin saturation A . HAV .
E) Serum TIBC B . Budd Chiari syndrome .
MED C . Wilson disease .
D . Hemochromatosis .
* A 34 years old female presents with vomiting E . Alpha one antitrypsin deficiency .
preceded by an headache of acute Onest . On
exam she was conscious , alert with photophobia * Each one of the following may raise ESR
but no neck stiffness . First brain CT scan is except ?
reported as normal . What is the most A . Female sex .
appropriate further management ? B . SLE .
A . CT brain with contrast . C . Polycythemia .
B . Repeat CT brain in 24 hours . D . Myeloma .
C . CSF examination . E . Increasing age .
D . Cerebral angiography .
E . MRI brain . * Where is the most common site for primary
cardiac tumors to occur in
* Most common causing microorganism of adult ?
spontaneous bacterial peritonitis ? A . Left atrium .
A . E coli . B . Left ventricle .
B . Streptococcus pneumonia . C . Left atrial appendage .
C . Listeria . D . Right ventricle .
D . Staphylococcus aureus . E . Right atrium .
E . Pseudomonas aeruginosa .
* All of the following can cause Hyperkalemia
* The best treatment of Wernicke Korsakoff except ?
syndrome is ? A . NSAIDs .
A . Thiamine . B . ACE inhibitors .
B . Folate . C . Spironolactone .
C . Cobaltammine . D . Furosemide diuretics .
D . Pyridoxine . E . Adrenal insufficiency .
E . Iron .
* All of the following are true about Long term
* Regarding secondary HTN , which is true ? oxygen therapy ( LTOT ) in
A . It account for 90% of the case's . COPD except ?
B . Addison disease is one of the causes . A . The goal is to maintaining SPo2 > 90% and
C . It can be easily controlled . PaO2 > 60 mmhg .
D . When associated with Hypokalemia , B . Main delivery devices include nasal cannula
hyperaldosteronism may be the cause . and venturi mask .
E . All patient should be investigated for C . Does not affect life expectancy in COPD
secondary causes at all ages . patient .
D . It is indicated in COPD who have PaO2 < 7.3
* A patient is seen in clinic complaining of kpa .
abdominal pain . Routine blood tests show Na = E . To get the benefits of LTOT , patients should
142 mmol /L , CL = 104 mmol/L , K = 4 mmol/L , breath supplemental oxygen for
HCO3 = 19 mmol/L , Ca = 2 mmol/L , PO4 = 1.5 at least 15 hours per day .
mmol/L , BUN = 28 mg/dl , creatinine = 112
mmol/L , Glucose = 180 mg /dl . What is the * All are risk factors for esophageal cancer
expected anion gap in mmol/L and except ?
‫ اﺳﺎﻣﻪ اﻟﺨﺰاﻋﻠﻪ‬: ‫اﻟﺪﻛﺘﻮر‬ ‫اﺳﺌﻠــــــــﺔ اﻟﺒﺎﻃﻨﻴــــــﺔ‬
Page 24
A . GERD . * Streptokinase therapy is recommended in acute
B . Achalasia . pulmonary embolism when ?
C . H pylori . A . Patient complains of shortness of breath .
D . Smoking . B . Patient complains of chest pain .
E . Alcohol . C . Blood pressure is 70/30 .
D . Pulmonary angiogram is positive .
E . Blood tinged sputum .
* All are causes of Pancytopenia except ?
A . Paroxysmal nocturnal hemoglobinuria . * All of the following are signs and symptoms of
B . Systemic lupus erythematosus . Acromegaly except ?
C . Megaloblastic anemia . A . Galactorrhea .
D . Iron deficiency anemia . B . Arthralgia .
E . Lymphoma . C . Excessive sweating .
D . Sleep apnea .
* In curative splenectomy , best time to give E . Hypergonadism .
pneumococcal vaccine ?
A . First week before surgery . * The best combined drugs for management of
B . 4 weeks before surgery . patient with HTN and history of MI is ?
C . Immediately after surgery . A . Thiazide and Amlodipine .
D . 2 weeks after surgery . B . Thiazide and virapamil .
E . At least one month after surgery . C . Atenolol and Ramipril .
D . Valsartan and Ramipril .
* A 25 years old woman complains of shortness E . Thiazide and Ramipril .
of breath after 10 hours of travel by air . RR = 30
BPM , BP = 110/70 , HR = 100 BPM . PE is * Causes of bilateral hilar lymphadenopathy are
suspected . The optimal duration of warfarin all of the following except ?
therapy is ? A . Sarcoidosis .
A . 6 weeks . B . Amyloidosis .
B . 3 months . C . Tuberculosis .
C . 6 months . D . Lymphoma .
D . 1 year . E . Extrinsic allergic alveolitis
E . Life long .
* All of the following are signs and symptoms of
* Woman with family history of cancer is at risk hyperparathyroidism primary type except ?
of all of the following except ? A . Confusion .
A . Endometrial cancer . B . Anorexia .
B . Ovarian cancer . C . Polyuria .
C . Colon cancer . D . Hypotension .
D . Gastric cancer . E . Short QT interval .
E . Breast cancer .
* All of the following are true about
* Numbers of death's with TB in one year over , pneumocystis carinii pneumonia except ?
the number of population in the middle of the A . It is caused by yeast like fungus
year is called ? pneumocystis jiroveci .
A . Mortality rate . B . It is seen in people with cancer ,
B . Crude death rate . chemotherapy , HIV and drug abusers .
C . Specific death rate . C . The risk of pneumocystis carinii pneumonia
D . Specific death ratio . increases when CD4 T_cell level
E . Case mortality rate . are less than 300 cells / ml .
D . X_Ray shows wide spread bilateral
* Site of action of Spironolactone is ? opacification with low grade fever , non
A . Proximal convoluted tubules . productive cough , loss of weight and night
B . Loop of henle ascending part . sweats .
C . Distal convoluted tubules . E . In HIV patient drug of choice is co _
D . Loop of henle descending part . trimoxazole .
E . Juxtaglomerular apparatus .

‫ اﺳﺎﻣﻪ اﻟﺨﺰاﻋﻠﻪ‬: ‫اﻟﺪﻛﺘﻮر‬ ‫اﺳﺌﻠــــــــﺔ اﻟﺒﺎﻃﻨﻴــــــﺔ‬


Page 25
* Patient with atrial fibrillation , which one of the C . SLE .
following factors would make a rate control D . Alpha methyldopa , quinidine , ciprofloxacin .
strategy rather than rhythm control more suitable E . Primaquine , sulfasalazine .
A . CHF .
B . Age > 65 years .
C . First presentation .
D . Symptomatic .
E . Atrial fibrillation secondary to a corrected
precipitant .

* All of the following is considered correct in the * All of the following are true about extrinsic
management of variceal bleeding except ? allergic alveolitis except ?
A . Patient should be resuscitated with IV fluid . A . Caused by hypersensitivity to inhaled organic
B . Monitoring of vital signs is a must . dusts .
C . Octreotide agonist should be added if variceal B . Type 2 or type 4 hypersensitivity .
bleeding if suspected . C . Pulmonary function test show restrictive loss
D . There is no need to add antibiotics for the of lung function .
treatment management GI bleeding caused by D . CBC show significant eosinophilia .
esophageal varices . E . Lymphocytic infiltration of alveoli and intra
E . TIPS procedure is an option. alveolar septa on lung biopsy .

* All of the following are true about Mycoplasma * Most common cause of upper GI bleeding is ?
pneumonia except ? A . Esophageal varices .
A . Autoimmune hemolytic anemia IgM . B . Gastric ulcer .
B . Ceftriaxone is the best treatment . C . Duodenal ulcer .
C . Most common primary atypical pneumonia . D . Erosive gastritis .
D . Bilateral lung consolidation with minimal E . Mallory weiss syndrome .
symptom .
E . Walking pneumonia and can cause epidemics * Diabetes mellitus diagnostic criteria include 2
hour glucose tolerance test , fasting glucose ,
* All of the following are true matching except ? and HbA1c respectively ?
A . Loud S2 in cor pulmonale and systemic HTN . A . > 140 mg/dL , > 110 mg/dL , > 6% .
B . Sof S2 in pulmonary stenosis , aortic B . >= 200 mg /dL , > 110 mg/dL , > 6.5% .
stenosis , and aortic regurgitation . C . >=200 mg/dL , > =126 mg/dL , >6% .
C. S3 is normal in youth , athletes , pregnants and D . >=200mg/dL , >=126 mg/dL , >=6.5% .
common pathological causes E . >=180 mg/dL , >=125 mg/dL , >=6.5% .
are LVF and mitral regurgitation .
D . Fixed splitting of S1 is heard with ASD . * ECG sign of hypothermia is ?
E . Loud S4 is sign of pathological state always A . Tachycardia .
like LVF , HTN , aortic stenosis . B . U wave .
C . Long QT interval .
* All are true about immune thrombocytopenic D . Short PR interval .
purpura except ? E . Peaked T wave .
A . Peak age 2 _ 6 years old and in adult 20 _ 40
years old . * All are causes of finger clubbing except ?
B . Minimal bruising and mucocutaneous bleed . A . Mesothelioma .
C . Splenomegaly and lymphadenopathy . B . Crohn disease .
D . Bone marrow aspirate shows increased C . Graves disease .
Megakaryocytes . D . COPD .
E . Non urgent treatment include corticosteroids , E . Idiopathic pulmonary fibrosis .
intravenous immunoglobulins , splenectomy ,
immunomodulators and no role of platelet * Which of the following make you suspect acute
transfusion . rather than chronic failure of kidneys ?
A . Anemia .
* All are causes of warm hemolytic anemia B . History of nephrotoxic drug .
except ? C . Small kidneys size .
A . CLL . D . Renal osteodystrophy .
B . Lung cancer . E . Peripheral neuropathy .
‫ اﺳﺎﻣﻪ اﻟﺨﺰاﻋﻠﻪ‬: ‫اﻟﺪﻛﺘﻮر‬ ‫اﺳﺌﻠــــــــﺔ اﻟﺒﺎﻃﻨﻴــــــﺔ‬
Page 26
* Which of the following contains the highest
* All are causes of prerenal azotemia except ? calories contents ?
A . HTN A . Butter .
B . GI bleeding B . Bread .
C . CHF C . Meat .
D . Sepsis D . Sugar .
E . Liver cirrhosis E . Macaroni .

* 25 years old known case of tuberculosis on * Iron deficiency anemia can cause all of the
treatment , drop of INR from 3 to 1.2 , which of following except ?
the following drugs responsible for this ? A . Koilonychia .
A . Rifampin B . Onycholysis .
B . Isoniazid C . Atrophic glossitis .
C . Ethambutol D . Stomatitis .
D . Pyrazinamide E . Pica .
E . Streptomycin
* Causes of Hyperkalemia all of the following
* 25 years old tall man had suddenly rapid and except ?
shallow breathing . In left CXR found to have 2 A . Cyclosporine .
cm pneumothorax . What is the best next step ? B . Atenolol .
A . Needle aspiration . C . Digoxin .
B . Chest tube . D . Triamterene .
C . Intubation . E . Ethacrynic acid .
D . O2 therapy and observation .
E . Thoracotomy . * All are true about renal tubular acidosis except
A . Type 1 and type 4 affect distal convoluted
* Best time to take simvastatin is ? tubules .
A . At bed time . B . Type 2 affect proximal convoluted tubules .
B . Morning . C . Type 1 and type 2 associated with
C . Before each meal . Hypokalemia , but type 4 associated with
D . In the evening . Hyperkalemia .
E . After each meal. D . Nephrocalcinosis associated with type 1 ,
Fanconi syndrome associated with type 2 , Renal
* Cyclosporine side effects include all of the insufficiency associated with type 4 .
following except ? E . Type 4 RTA associated with
A . Gingival hyperplasia . hyperaldosteronism .
B . Alopecia .
C . Hyperglycemia . * A patient has a potassium level of 7 mmol/L .
D . Jaundice . Which of the following
E . Tremor . intervention is priority ?
A . Administer a laxative .
* All are true about cystic fibrosis except ? B . Administer a loop diuretics Furosemide .
A . Cor pulmonale C . Administer Spironolactone .
B . Rectal prolapse D . Prepare the patient for dialysis and place the
C . Abdominal aortic aneurysm patient on cardiac monitor .
D . Metabolic Alkalosis E . Give patient insulin and glucose .
E . Gallstones
* Incidence rate is defined as ?
* Activated charcoal is used in one of the A . Number of old case's in a population per unit
following poisoning ? of person _ time .
A . Aminophylline . B . Number of new case's in a population per unit
B . Lead . of person _ time .
C . Iron . C . Number of all case's in a population over a
D . Ethylene glycol . defined period of time .
E . Lithium . D . Number of old and new case's in a population
per unit of person _ time .
‫ اﺳﺎﻣﻪ اﻟﺨﺰاﻋﻠﻪ‬: ‫اﻟﺪﻛﺘﻮر‬ ‫اﺳﺌﻠــــــــﺔ اﻟﺒﺎﻃﻨﻴــــــﺔ‬
Page 27
E . Number of new case's in a population per unit 2- It is contraindicated in patient with Guillain
of person over a defined period of one year . Barre syndrome
3- It is composed of three strains of influenza
* All of the following are Quantitative tests virus
hypothesis except ? 4- It is contraindication in immunocoprpmised
A . Why ? patients
B . What ? 5- It is safe in pregnancy
C . How much ?
D . How many ?
E . When ?
* All of the following are criteria for major * Which of the following is ture concerning sickle
depression except ? cell trait:
A . Decreased mood . 1- It is a cause of anemia
B . Decreased pleasure . 2- It protects against malaria
C . Hypersomnia . 3- It occurs mainly in female
D . Hallucinations . 4- It is a cause of frequent sikle cells in the
E . Suicidal ideation . peripheral blood
5- It is associated with hemoglobin H
* A 70 year old woman with a few months history
of leg pain ,gum bleeding and petechiae on the * Male was admitted with pneumonia caused by
arm after the application of a mycoplasma,all of the following are
sphygmomanomerter.which of the following characteristics of this type of pneumonia except:
vitamin defficiencies dose pateint have ? 1- Insidious onset of the disease
1- Vit A 2- Fever of 38 C
2- Vit B 3- Sever leukocytosis
3- Vit C 4- Good response to macrolides
4- Vit B 12 5- Hemolytic anemia
5- Vit E
* All of the following are considered anti-
* Regarding detection of H-pylori infection,the pseudomanal antibiotics except:
least useful test is : 1- Ceftazidime
1- Tissue culture of gastric biopsy 2- Ciprofloxacin
2- Urea breath test 3- Ceftriaxone
3- Histology of gastric biopsy 4- Gentamicin
4- Rapid urease test of gastric biopsy 5- Cefipime
5- Stool antigen test
* The activities of school health are except:
* Which of the following antibiotic has been 1- Screening of school children to detect and
particularly assciated with hemolytic disease are abnormality
reactions in patient with G6PD ? 2- Teaching students about health system
1- Amoxicillin 3- Providing treatment any student falls sick in
2- Clindamycin the school
3- Vancomycin 4- Health education
4- Cloxacillin 5- Immunization
5- Trimethoprim/sulfamthoxazole
* One of the following is not a systemic
* Which of the following antibiotics is absolutely complication of ulcerative colitis:
contraindication in pregnant 1- Erythema nodosum
wowen: 2- Pyoderma gangrenosum
1- Imipenem 3- Arthritis
2- Azithromycin 4- Malabsorption
3- Ceftriaxone 5- Sclerosing cholangitis
4- Cloxacillin
5- Tetracycillin * Inflammatoy diarrhea is characterized :
1- Absence of leucocytes in the stool
* All the following are true about influenza 2- Absence of fever
vaccine except: 3- Presence of leucocytes in the stool
1- It is given annually 4- Absence of blood in the stool
‫ اﺳﺎﻣﻪ اﻟﺨﺰاﻋﻠﻪ‬: ‫اﻟﺪﻛﺘﻮر‬ ‫اﺳﺌﻠــــــــﺔ اﻟﺒﺎﻃﻨﻴــــــﺔ‬
Page 28
2- Controlling blood glucose levels
* The route of transmission of this virus is fecal 3- Smoking cessation
oral route: 4- Use of an ACE inhibitor
1- Hepatitis B virus 5- Use of aspirin
2- Hepatitis C virus
3- Hepatitis E virus
4- Hepatitis D virus
5- Human immune deficiency virus

* In the treatment of infective endocarditis ,all * Which of the following is most sensitive early
true except: sign for detecting diabetic nephropathy:
1- Persistent fever may indicate a nonbacterial 1- Serum creatinine
like fungal cause 2- urinary albumin/creatinine ratio
2- Cidal drugs rather than statics should be used 3- Serum urea level
3- Always await the culture results before giving 4- Ranal ultrasonography
any teatment 5- Creatinine clearance
4- Inrtvenous route is always perferred
5- Persistent fever may indicate a drug fever * The following are clinical features os restrictive
cardiomyopathy:
* The most common cause of upper 1- Presentation which mimics constrictive
gastrointestinal bleeding: pericarditis
1- Esophageal varices 2- Primarily characterized by impaired diastolic
2- GERD function
3- Peptic ulcer disease 3- Association with primary or secondary
4- NSAID induced erosive gastritis amyloidosis
5- Esophageal ulcer 4- Complication of conditions inducing marked
peripheral blood eosinophilla
* One of the following is not a presentation of 5- Gross cardiomegaly on CXR
liver cirrhosis
1- Gynecomatia * The risk of transmission of hepatitis B virus
2- Abdomianl collateral circulation followingpercutaneous needle stick injury is:
3- Asterixis 1- 0.3%
4- Cerebral edema and brain hernation 2- 3%
5- Ascites 3- 30%
4- 60%
* The diagnosis of live cirrhosis is established by: 5- 90%
1- Abdominal U/S
2- Prothrombin time and serum albumin * 70 year old man presents for health
3- Liver biopsy assessment. which Bp reading would indicate
4- Testing for HbsAg ,HCV Ab, ANA, iron and the highest risk for a cerebrovascular accidents:
urine cupper 1- 110/90
5- Platelets count to todoucumenthypersplenism 2- 120/90
3- 130/70
* In the coure of DKA ,which of the following is 4- 140/80
true about serum potasstum level: 5- 155/70
1- It remain unaffected
2- It can appear normal but total body potassium * All the following about Hepatitis A infection true
may actually be low except:
3- It can appear normal but total body potassium 1- Fecal oral transmission
may actually be high 2- It is a common cause of chronic liver disease
4- It will naturally be corrected by insulin 3- Mortality increases with advancing age
administration 4- Maximum infectivity of the disease is before
5- None of the above the onset of jaundice
5- Hepatitis A vaccine is protective from the
* The most effective method for preventing disease
diabetic retinopathy is :
1- Reducing triglyceride level
‫ اﺳﺎﻣﻪ اﻟﺨﺰاﻋﻠﻪ‬: ‫اﻟﺪﻛﺘﻮر‬ ‫اﺳﺌﻠــــــــﺔ اﻟﺒﺎﻃﻨﻴــــــﺔ‬
Page 29
* In patient with macrocytic anemia,which one A. It's due to deficiency of cortisol hormone .
the following features B12 B . Enlarged extremities are commonly seen .
defieciency rather than folate deficiency : C . Osteoporosis is not a feature .
1- Mild jaundice D . Iatrogenic cause is uncommon .
2- Loss of weigh E . A moon face and buffalo hump are
3- Skin purpura characteristic .
4- Lower limb neuropathy
5- Recurrent throat infection

* Peripheral blood neurophil leucocytosis would * In hypercapnia are all true except ?
be an expected finding in all of A . Bounding pulse
the following except: B . Headache
1- Connective tissue diseases C . Peripheral vasoconstriction
2- Corticosteroid therapt D . Confusion
3- Pregnancy E . Flapping tremor
4- Whooping cough
5- Mesenteric infarction * To assist in the decision whether to hospitalize
a patient with community acquired pneumonia ,
* Regarding santitation barriers all the following each of the following may be a factor in favor of
are true except: hospitalization except which one ?
1- Protection of water supply A. The patient is confused
2- Protection of food B . Serum creatinine > 2.0 mg / dl
3- Personal hygiene C . Respiratory rate > 30
4- Control of flies D . Blood pressure < 90 mg hg
5- Using of spring water better than tap water E . Age > 64 years

* All the following are true about Hepatitis B virus * All of the following can be used in hypertensive
vaccine except: crisis except ?
1- It is given in three doses A. Sodium nitroprusside
2- Common schedule is: 0, 1 , 6 months B . Furosemide
3- It is safe in immunocompromised individuals C . Labetalol
4- Booster dose is routinely given after 10 years D . Clevidipine
from finishing the series(the three doses) E . Hydralazine
5- It is highly protective to hepatitis B virus
* Diarrhea may occur with all of the following
* Which of the following features suggests except ?
adrenal insufficiency : A . Food poisoning
1- Hypoglycemia B . Viral gastroenteritis
2- Hypokalemia C . Inflammatory bowel disease
3- Leukopenia D . Colonic Neoplasia
4- Anemia E . Hypothyroidism
5- Hypertension
* A 7 years old has severe microcytic anemia due
* 55 year old asymptomatic obese female to B Thalassemia major ( homozygous ) . He
presents to your clinic for checkup and requires frequent blood transfusion( once every
general advice,youadvice her all of the following sex week's ) To prevent the skeletal and
condition are strongly assciated with obesity developmental complications of thalassemia .
except one: Which of the following medication is also
1- Insulin resistance indicated in the treatment of patients
2- Hypertension requiring frequent blood transfusion ?
3- Osteoporosis A . Oral calcium supplement
4- Arteriosclerosis B . Fresh frozen plasma
5- Hypoventilation C . Desferrioxamine
D . Pencillamine
* Which of the following is true about Cushing E . Cryoprecipitate
syndrome ?
‫ اﺳﺎﻣﻪ اﻟﺨﺰاﻋﻠﻪ‬: ‫اﻟﺪﻛﺘﻮر‬ ‫اﺳﺌﻠــــــــﺔ اﻟﺒﺎﻃﻨﻴــــــﺔ‬
Page 30
* One of the following vaccines is a killed vaccine C . Obesity and HTN are non modifiable risk
A . Measles vaccine factor .
B . Mumps vaccine D . Creatinine kinase can be raised due to other
C . Rubella vaccine non cardiac cause .
D . Pertussis vaccine E . Cardiac troponin remains elevated for 24 hour
E . Oral polio vaccine only .

* All of the following can cause Hemoptysis


* All of the following are found in patient with except ?
bronchial asthma except ? A . Pneumonia
A . Hyperinflated chest B . Idiopathic pulmonary fibrosis
B . Wheezing C . Tuberculosis
C . Dyspnea D . Pulmonary embolism
D .Clubbing E . Goodpasture syndrome
E . Cough
* Regarding HTN which is true ?
* Regarding Sarcoidosis which is true ? A . The patient should be screened for secondary
A . It is a caseating Granuloma HTN .
B . It has no extrapulmonary manifestation . B . Secondary HTN account for 90% of cases .
C .Spontaneous resolution may occur in 2 year . C . Addison disease is one of the causes of
D .Steroids is always indicated . secondary HTN .
E . Increase in serum ACE level is diagnostic . D . 24 hr urinary collection for metanephrin level
may be diagnostic .
* All of the following causes exudative pleural E . In isolated systolic HTN no treatment is
effusion except ? required .
A .Constrictive pericarditis
B .Mesothelioma * A 35 year old is evaluated for symptoms of
C .Rheumatoid arthritis S.O .B . Her reports no other lung or heart
D .Pneumonia disease . He smokes half pack a day for the past
E .Tuberculosis 10 years . On examination his JVP is 2 cm , heart
sound normal . Lung clear . CXR shows
* Atrial fibrillation can be caused by all the hyperinflation and Emphysematous changes ,
following except ? while pulmonary function tests shows an FEV1 /
A . Mitral valve disease FVC ratio < 70% . Which of the following is most
B . HTN likely diagnosis ?
C . Hypothyroidism A . Alpha one antitrypsin deficiency
D . Pneumonia B . B glucosidase deficiency
E . Atrial myxoma C .G6PD
D .Glucocerebrosidase deficiency
* A 73 years old man presents with several E . Growth hormone deficiency
episodes of hematemesis . Examination shows
signs of orthostatic hypotension and melena . * Sacroilietis commonly occurs in all of the
What is the first priority in caring for this patient ? following diseases except ?
A .NG tube placement and gastric lavage . A . Ulcerative colitis
B . Resuscitation with adequate IV access and B . Crohn disease
appropriate fluid and blood product fusion . C . Gout
C .IV infusion of H2 receptor antagonist to stop D . Ankylosing spondylitis
bleeding . E . Psoriatic arthritis
D .Urgent upper endoscopy .
E . Urgent surgical consultation . * The following are more in favor of type one DM
than type two except ?
* Regurgitation AC syndrome one of the A . Association with ketoacidosis .
following is true ? B . Association with HLA_DR3 or HLA_DR4 .
A . Cardiac enzymes is always elevated . C . Strong family history of diabetes .
B . ECG changes is mandatory for the diagnosis . D . Present of islet cell antibodies .
‫ اﺳﺎﻣﻪ اﻟﺨﺰاﻋﻠﻪ‬: ‫اﻟﺪﻛﺘﻮر‬ ‫اﺳﺌﻠــــــــﺔ اﻟﺒﺎﻃﻨﻴــــــﺔ‬
Page 31
E . Abrupt onset of signs and symptoms . B . Reassurance
C . Barium esophagrame
* Regarding bronchial asthma all are true except D . PPI therapy for 6_8 week's
A . It is chronic airway inflammation . E . Abdominal CT
B . Treatment with long acting B2 agonist is the
cornerstone . * Causes of pancytopenia includes all the
C . PFT increase in FEV1 more than 200 ml and following except ?
12 % indicates reversibility . A . Megaloblastic anemia
D . It's characterized by dyspnea , chest B . Aplastic anemia
tightness , cough and wheezes . C . Myelofibrosis
E . Attack can be induced by exercise D . IDA
* Signs and symptoms of hypothyroidism include E . Acute leukemia
all the following except ? * Which of the following is true of psoriatic
A . Thick dry skin arthropathy ?
B . Depression A . Nail pitting may give a clue to the diagnosis .
C . Constipation B . Absence of plaques of psoriasis excludes the
D . Tachycardia diagnosis .
E . Weight gain C . The diagnosis can be confirmed by a blood
marker .
* A 65 years old man develops palpitations and D . Typhi can some times be seen over the
dizziness . His blood pressure is 80/50 mm hg affected joints .
and his pulse is regular at 150 / min . His ECG E . Severity is related to skin changes severity .
shows a saw toothed pattern of P waves . Which
of the following procedure is most appropriate in * Regarding Nephrotic syndrome all are true
converting him back to sinus rhythm ? except ?
A . Carotid sinus pressure A . Renal vein thrombosis is a complication .
B . Gagging procedure B . Increase bleeding tendency
C . Valsalva maneuver C . Increase susceptibility to infection .
D . Eyeball compression D . DM is a recognized cause .
E . Electrical cardioversion E . Hypoalbuminemia and Proteinuria is present

* A 23 years old woman experienced watery * 60 years old male presented with massive
diarrhea , nausea, vomiting and abdominal hematemesis , examination revealed
cramps 6 hours after eating a salad and a Splenomegaly , clubbing and Palmar Erythema .
hamburger in a local restaurant . The most likely The cause of his hematemesis ?
organism causing her disease is ? A . Mallory Weiss syndrome
A . Vibrio vulnificus B . Gastric ulcer
B . Listeria monocytogenes C . Angiodysplasia
C . Yersinia enterocolitica D . Esophageal varices
D . Clostridium difficile E . Gastritis
E . Staphylococcus aureus
* A 63 years old man is involved in a motor
* Air conditioner repair worker is more likely to be vehicle accident and is brought to the hospital .
at risk of contracting ? On examination, his blood pressure is 90/60 mm
A . Hepatitis hg , pulse 110 / min , and his abdomen is
B . Brucellosis distendes and rigid , he has internal injuries and
C . Legionnaires disease bleeding on an abdominal CT scan , and requires
D . Histoplasmosis emergent blood transfusion on his way to the
E . Tuberculosis operating room . His blood group is AB , which of
the following statement is not correct ?
* A 19 year old woman present with daily acid A . He is a universal recipient .
reflux symptoms and intermittent dysphagia to B . He has anti A and anti B in his serum .
solid . Since onest of symptoms 3 months ago , C . If a cross match is not available, group O
she has lost 2 kg , she has sporadic rectal RBCs are universal .
bleeding . Her past medical history is negative . D . If insufficient AB blood is available , type A
Her father had stomach cancer at age 42 . What can be used .
would you recommend ? E . If insufficient AB blood is available , type B
A . EGD can be used .
‫ اﺳﺎﻣﻪ اﻟﺨﺰاﻋﻠﻪ‬: ‫اﻟﺪﻛﺘﻮر‬ ‫اﺳﺌﻠــــــــﺔ اﻟﺒﺎﻃﻨﻴــــــﺔ‬
Page 32
Steroids are the cornerstone treatment .
* The study of the distribution of a disease or It is a curable disease .
condition in a population is PFT reversibility always present .
referred to as ? Can cause right sided heart failure .
A . Descriptive
B . Analytical * All of the following are cause of chronic cough
C . Experimental except ?
D . Prospective A . GERD
E . Retrospective B . Sodium valproate
C . ACE_i
D . Long term smoking
E . Postnasal drip
* All about irritable bowel syndrome are true * Which of the following antiarrhythmic drug
except ? mediates it's effect by interfering with movement
A . Abdominal pain relieved by defecation . of calcium through the slow channel ?
B . Symptoms can be increased by stress . A . Phenytoin
C . Waking at night with pain or diarrhea support B . Verapamil
the diagnosis . C . Lidocaine
D . Altered bowel habit is common . D . Amiodarone
E . Examination is often normal . E . Bretylium

* Regarding diabetic ketoacidosis which is true ? * 63 years old woman is seen in the emergency
A . Dehydration is less life threatening than room with acute shortness of breath . There is no
hyperglycemia . history of heart or lung problems in the past . She
B . Potassium level initially is high and falls down was recently diagnosed with breast cancer and is
with treatment . under going active treatment . On examination
C . Is a late complication of type one DM . her blood pressure is 120/80 mm hg , pulse 100 /
D . Can be managed as outpatient in most of the min , and heart and lungs are normal . There are
cases . no clinical sings of DVT . Which of the following
E . Elevated WBC count always indicates investigation is most likely to rule out a PE ?
infection . A . Normal CXR
B . Normal ECG
* All of the following are side effects of steroids C . Normal ventilation _ perfusion lung scan .
except ? D . Normal high resolution CT
A . Elevated blood pressure E . Normal MRI
B . Acne
C . Skin bruises * A 66 year old man has progressive shortness of
D . Hypoglycemia breath due to COPD . He is currently able to do
E . Cataract his activities of daily living , but has trouble
walking more than 50 m . His physical
* Blood gas analysis in type one respiratory examination reveals hyperinflation , increased
failure shows ? resonance to percussion , and bilateral expiratory
A . High PCO2 and normal PO2 . wheezes . He is in appropriate medical therapy
B . Low PCO2 and normal PO2 . for his stage of COPD . Which of the following is
C . Normal PCO2 and high PO2 . also indicated in the management of this
D . High PCO2 and low PO2 . condition ?
E . Normal PCO2 and low PO2 . A . Meningococcal vaccine
B . Yearly influenza vaccine
* In salmonellosis which is true ? C . Weight reduction obese
A . Positive blood culture occur in the second D . Haemophilus influenza B vaccination
week . E . Pneumococcal vaccination
B . Positive stool culture occur in the first week .
C . Splenomegaly doesn't occur . * A 39 year old woman with HTN , is taking
D . Can be complicated by Osteomyelitis . hydrochlorothiazide ,enalapril and diltiazem . She
E . The route of transmission is air borne . desires to become pregnant . The most
important step would be to ?
* One of the following is true about COPD ? A . Switch HCTZ to Furosemide
It is one of the cause restrictive PFT . B . Switch diltiazem to amlodipine
‫ اﺳﺎﻣﻪ اﻟﺨﺰاﻋﻠﻪ‬: ‫اﻟﺪﻛﺘﻮر‬ ‫اﺳﺌﻠــــــــﺔ اﻟﺒﺎﻃﻨﻴــــــﺔ‬
Page 33
C . Discontinue diltiazem
D . Discontinue enalapril * Skin cancer result from exposure to one of the
E . Discontinue HCTZ following ?
A . Infrared rays
* Regarding Acromegaly all are true except ? B . X rays
A . Caused by tumor in adrenal gland in more than C . Solar ultraviolet
90 % . D . Ionizing radiation
B . Excessive sweating . E . Microwave
C . Large hand and feet .
D . Increased growth hormone .
E . Proganthism .

* All are features of LVF except ?


A . Orthopnea * Regarding immunoglobulins all are true except?
B . Hepatomegaly A . Rhesus antibodies are of IgG class .
C . Pulsus alternance B . IgG is the first response to invading organism
D . Bilateral basal crepitation C . IgA has the highest concentration in human
E . Kerly B line on CXR secretion .
D . IgM is the best complement activator .
All of the following statement are correct about E . IgE mediates allergy by binding antigen on the
routine urine examination except ? membrane of the cell .
A . Specific gravity of 1015 is normal .
B . RBCs is a normal finding . * Social class can be measured by all of the
C . One RBC is a normal finding. following except ?
D . One WBC is a normal finding. A . Sex
E . Negative sugar is a normal finding B . Income
C . Education
* Which oxygen delivery device should be used in D . Occupation
COPD patient ? E . Residency
A . Trauma mask
B . Nasal cannulae * One of the following disease is considered
C . Simple face mask eradication worldwide since more than 30 years
D . Venturi mask A . Smallpox
E . Nonrebreather mask B . Chickenpox
C . Plague
* One of the following can cause Hypokalemia ? D . Typhus
A . Spironolactone E . Mumps
B . Hyperaldosteronism
C . ACE_i * All of the following sentences are true about
D . Addison disease the assessment of bleeding except ?
E . Renal tubular acidosis type 4 A . Taking blood sample at the time of bleeding
give an accurate level of HB .
* Which of the following most likely to cause B . Hypovolemia increases the risk of tissue
deviation of trachea ? hypoxia and multiorgan failure .
A .Previous TB of the right upper lobe C. Blood loss could be estimated from the size of
B .Small right pleural effusion blood clot and the weight of
C .A Left basal pneumonia swabs use .
D . Idiopathic pulmonary fibrosis D . Any thirsty patient should be considered
E . Blunted right costophrenic angle Hypovolemic .
E . Arterial blood gas is a quick method to
* Optimal medical therapy in patient with carotid measure Hb level .
artery stenosis ?
A . Smoking cessation * Epidemiology mean ?
B . Antiplatelet agent A . Study of population ( study of the distribution
C . Control HTN and determination health related states or even .
D . Statin drugs B . Study of disease only .
E . All are true C . Study population growth .
‫ اﺳﺎﻣﻪ اﻟﺨﺰاﻋﻠﻪ‬: ‫اﻟﺪﻛﺘﻮر‬ ‫اﺳﺌﻠــــــــﺔ اﻟﺒﺎﻃﻨﻴــــــﺔ‬
Page 34
D . Study of mortality . the following laboratory values would be most
E . Study of chronic disease . consistent with a prerenal etiology of his renal
insufficiency ?
* Patient with cystic fibrosis expected to have A . FEna of 3% .
deficiency in all of the following except ? B . Urinary sodium level of 10 mEq/L .
A . Vitamin A C .CVP reading of 10 mmHg .
B . Vitamin B12 D . Gentamycin trough level of 4uq/ml .
C . Vitamin K
D . Vitamin D
E . Vitamin E

* A 22 year old woman presents with fatigue,


* Regarding Mycoplasma pneumonia all are true arthralgias , and a nagging dry cough for the past
except ? 6 weeks , but no shortness of breath . On
A . It is one of atypical pneumonia . physical examination, her lungs are clear to
B . Can cause autoimmune Hemolytic anemia . auscultation , and she has bilateral pretibial
C . Can cause meningioencephalitis . tender Erythematous raised nodules . Which of
D . Cephalosporin antibiotic is a drug of choice . the following is your
E . Bilateral patchy consolidation . best next step ?
A . Chest radiograph .
* One of the following is most suited for B . CT chest .
detection of diabetic ? C . Empiric treatment for Postnasal drip .
A . Renal ultrasound D . Antinuclear antibody .
B . Urine analysis for cast E . Initiation of antituberculosis therapy .
C . Urine albumin
D . IVP * An obese 50 year old man with a history of
E . Serum creatinine asthma returns with complaints of occasional
dyspepsia and nocturnal cough . He wakes up in
* A 30 year old woman is noted to have blood the morning with a sour taste in his mouth . His
pressures in the 160/100 mmHg range . She also current medications include inhaled
has increased obesity , especially around her corticosteroids and a short acting B2 agonist .
abdomen , with striae . She has been bruising Which of the following should be your next step ?
very easily and has hirsutism . Which of the A . 24 hour esophageal PH monitoring .
following is the most likely diagnosis ? B . Chest radiograph .
A . Hyperthyroidism C . Initiation of omeprazole .
B . Hyperparathyroidism D . Short course of oral corticosteroids .
C . Coarctation of aorta E . Initiation of allergy desensitization
D . Cushing syndrome
E . Pheochromocytoma * A patient with known asthma undergoing
therapy with inhaled corticosteroids and
* A 45 year old man with type two diabetes is intermittent short acting B2 agonist presents
noted to have blood pressures of 145/90 and with complaints of nocturnal awakening
150/96 mmHg on two separate occasions . secondary to cough and occasional wheezing .
Which of the following is the best initial therapy This episode occurs three to four times per
for this patient ? week . Pulmonary function tests in the past have
A . Hydrochlorothiazide . shown mild obstructive lung disease . Which of
B . ACE_i . the following is the best next step ?
C . B blockers . Oral steroids
D . CCB . Leukotriene inhibitors
E . B blockers and Hydrochlorothiazide . Long acting B2 agonist
Theophylline
* A 62 year old diabetic man underwent an Antireflux therapy
abdominal aortic aneurysm repair 2 days ago .
He is being treated with Gentamycin for a UTI . * Which of the following is most accurate ?
His urine output has fallen to 300 ml over 24 . Cough caused by Captopril may resolve with
hours , and his serum creatinine has risen from switching to enalapril .
1.1 mg/dl on admission to 1.9 mg/dl . Which of
‫ اﺳﺎﻣﻪ اﻟﺨﺰاﻋﻠﻪ‬: ‫اﻟﺪﻛﺘﻮر‬ ‫اﺳﺌﻠــــــــﺔ اﻟﺒﺎﻃﻨﻴــــــﺔ‬
Page 35
. Initial treatment of a chronic cough should * A 56 year old man complaining of fatigue and
include codeine or a similar opiate derivative to weight loss . He has never had any health
suppress the cough . problems , but he has smoked a pack of
. Cough caused by reflux can be effectively ruled cigarettes per day for about a 35 years . skin
out by a negative history of heartburn or hyperpigmentation , and digital clubbing . He
dyspepsia . appears euvolemic . The sodium level is 126
. More than one condition often is responsible for mEq/L , potassium level is 6.7 mEq/L , creatinine
causing a chronic cough in a given patient . level is normal, and bicarbonate and chloride
levels are low . Which of the following is the likely
cause of his Hyponatremia given his
presentation
SIADH
Hypothyroidism
* A 55 year old man is noted to have moderately Gastrointestinal losses
severe congestive heart Adrenal insufficiency
failure with impaired systolic function . Which of Renal insufficiency
the following drugs would * An 83 year old woman comes to your office
most likely lower his risk of mortality ? complaining of a headache and mild confusion .
ACE_i Her medical history is remarkable only for HTN ,
Loop diuretics which is well controlled by Hydrochlorothiazide .
Digoxin Her examination and laboratory tests show no
Aspirin signs of infection , but her serum sodium level is
119 mEq/L , and plasma osmolarity is 245
* A 35 year old woman is noted to have chest mOsm/kg . She appears to be clinically
pain with exertion , and has been passing out Hypovolemic . Which of the following is the best
recently . On examination she is noted to have a initial therapy ?
harsh systolic murmur . Which of the following is Fluid restriction
the best therapy for her condition ? Infusion of 0.9%saline
Coronary artery bypass Infusion of 3% saline
Angioplasty Infusion of 3% saline with Furosemide
Valve replacement
Carotid endarterectomy * A 58 year old man has undergone a lengthy
colon cancer surgery . On the first postoperative
* A 55 year old man is noted to have congestive day , he is noted to have significant
heart failure and states that he is comfortable at Hyponatremia with a sodium level of 128 mEq
rest but becomes dyspneic even with walking to /L . You suspect that the Hyponatremia is due to
bathroom . On echocardiography , he is noted to the
have an ejection fraction of 47% . Which of the intravenous infusion of hypotonic solution .
following is the more accurate description of this Which of the following laboratory findings
patients condition ? supports your diagnosis ?
Diastolic dysfunction Urine sodium > mmol/L
Systolic dysfunction Urine osmolality > 200 mOsm /L
Dilated cardiomyopathy Serum osmolarity < 280 mOsm /kg
Pericardial disease Serum potassium > 5 mEq/L

* A 24 year old man develops seizures following * Which of the following patients is most likely to
an emergent splenectomy after a car accident . be a candidate for bone mineral density
His serum sodium level is initially 116 mEq / L screening ?
and is corrected to 120 mEq/L over the next 3 A 65 year old, thin , white woman who smokes
hours with hypertonic saline . Which of the and is 15 years postmenopausal .
following factors most likely led to his B 40 year old white woman who exercises daily
Hyponatremia ? and still menstruates .
Elevation of serum vasopressin C healthy 75 year old white man who is
Administration of hypertonic solutions sedentary .
Volume depletion D 60 year old overweight black woman .
Seizures induced Hyponatremia E 35 year old asthmatic woman who took
prednisone 40 mg/d for a 2 week course 1 week
ago
‫ اﺳﺎﻣﻪ اﻟﺨﺰاﻋﻠﻪ‬: ‫اﻟﺪﻛﺘﻮر‬ ‫اﺳﺌﻠــــــــﺔ اﻟﺒﺎﻃﻨﻴــــــﺔ‬
Page 36
Factor 5 Leiden mutation
* During which of the following periods in a Antiphospholipid antibody syndrome
woman's life is the most bone mass Familial malignancy syndrome
accumulated
Ages 15 to 25 * A 54 year old woman is noted to have cervical
Ages 25 to 35 cancer and presents with significant vaginal
Ages 35 to 45 bleeding with a hemoglobin level of 7 g/dL . Her
Ages 45 to 55 left leg is swollen , which on Doppler
investigation reveals a DVT . Which of the
* A 60 year old woman presents with the results following is the best treatment for the
of her DEXA scan . She has a T score of _1.5 at thrombus ?
the hip and _2.5 at the spine . Which of the Intravenous unfractionated heparin .
following is the most accurate interpretation of Fractionated subcutaneous heparin .
these results ? Subcutaneous unfractionated heparin.
She has osteoporosis at the spine and osteopenia Oral warfarin ( Coumadin ) .
at the hip . Vena cava filter .
She has osteoporosis in both area's .
This is a normal examination .
She has osteoporosis of the hip and osteopenia
at the spine . * A 72 year old man develops severe pain and
You need to know the Z score . swelling in both knees , shortly after undergoing
an abdominal hernia repair surgery . Physical
* You see a 70 year old woman in your office for examination shows warmth and swelling of both
a routine checkup , and you order a DEXA scan knees with large effusions . Arthrocentesis of the
for bone mineral density screening . The T score right knee reveals the presence of intracellular
returns as 2.5 standard deviations SD in the and extracellular weakly positive birefringent
spine and _2.6 in the hip . Which of the following crystals in the synovial fluid . Gram stain is
statements is most accurate ? negative . Which of the following is the most
This patient has osteopenia . likely diagnosis ?
Estrogen replacement therapy should be started Gout
with an anticipated rebuilding of Septic arthritis
bone mass to near normal within 1 year . Calcium oxalate deposition disease
Swimming will help build bone mass . Reactive arthritis
Bisphosphonates would reduce the risk of hip Pseudogout
fracture by 50%
* A 65 year old man with a history of chronic
* A 35 year old woman complains of calf HTN , DM , and degenerative joint disease
tenderness and acute dyspnea . The arterial presents with acute onest of severe pain of the
blood gas reveals PO2 ( Partial pressure of metatarsophalangeal joint and swelling of the
oxygen ) of 76 mmHg . Which of the following is left first toe . Physical examination shows
the most common physical examination finding exquisite tenderness of the joint , with swelling ,
of pulmonary embolism ? warmth ,
Wheezing . and Erythema . The patient has no history of
Increased pulmonary component of the second trauma or other significant medical problems .
heart sound . Synovial fluid analysis and aspiration is most
Tachypnoea . likely to show which of the following ?
Calf swelling . Hemorrhagic fluid .
Pulmonary rales . Needle_ shaped , negatively birefringent crystals
Gram negative organisms .
* A 39 year old man is noted to have a DVT Noninflammatory fluid .
without any known risk factors . He notes that his Rhomboidal , positively birefringent crystals
brother also developed a pulmonary embolism at
age 45 years , and his mother developed a clot in * A 17 year old sexually active adolescent male
the leg when she was in her thirties . Which of the presents with a 5 day history of fever , chills , and
following is the most likely inherited disorder in persistent left ankle pain and swelling . On
this patient ? physical examination , maculopapular and
Protein S deficiency pustular skin lesions are noted on the trunk and
Antithrombin 3 deficiency
‫ اﺳﺎﻣﻪ اﻟﺨﺰاﻋﻠﻪ‬: ‫اﻟﺪﻛﺘﻮر‬ ‫اﺳﺌﻠــــــــﺔ اﻟﺒﺎﻃﻨﻴــــــﺔ‬
Page 37
extremities . He denies any symptoms of morning .She comes in now because she had
genitourinary tract infection . Synovial fluid fainted this morning . Which of the following is
analysis is most likely to show which of the the most likely cause of her syncope ?
following ? Carotid sinus Hypersinsivety
WBCs 75.000/mm3 with 95% polymorphonuclear PE
leukocytes . Autonomic neuropathy
RBCs 100.000/mm3 , WBCs 1000/mm3 . Critical aortic stenosis
WBCs 48.000/mm3 with 80% lymphocytes .
WBCs 500/mm3 with 25% polymorphonuclear * A 74 year old man with no prior medical
leukocytes . problems faints while shaving . He has a quick
recovery and has no neurologic deficits . His
* A 22 year old man presents with complaints of blood sugar level is normal , and ECG shows a
low back pain for 3 to 4 months and stiffness of normal sinus rhythm . Which of the following is
the lumbar area , which worsen with inactivity . the most useful diagnostic test of his probable
He reports difficulty in getting out of bed in the condition ?
morning and may have to roll out sideways , Carotid massage
trying not to flex or rotate the spine to minimize Echocardiogram
pain . A Lumbosacral spine X_Ray film would CT scan of head
most likely show which of the following ? Serial cardiac enzymes
Degenerative joint disease with spur
information .
Sacroiliitis with increased sclerosis around the
sacroiliac joints .
Vertebral body destruction with wedge fractures . * A 49 year old man is admitted to the intensive
Osteoporosis with compression fractures of care unit with a diagnosis of an inferior MI . His
L3_L5 . heart rate is 35 bpm and blood pressure 90/50
Diffuse osteonecrosis of the LS spine . mmHg . His ECG shows a mobitz type 1 heart
block . Which of the following is the best next
* A 36 year old woman was seen by her physician step ?
due to pain in her hands, wrists , and knees . She Atropine
is diagnosed with rheumatoid arthritis . Which of Transvenous pacer
the following treatments will reduce joint Lidocaine
inflammation and slow progression of the Observation
disease ?
NSAIDs * A 42 year old woman is being treated with
Joint aspiration infliximab for rheumatoid arthritis . After 6
Methotrexate months of therapy , she develops persistent
Systemic corticosteroids fever , weight loss , and night sweats , and
tuberculosis is suspected . Which of the
* An 18 year old adolescent female is brought to following is most likely location of the
the emergency room because she fainted at a tuberculosis ?
rock concert . She apparently recovered A Middle and lower lung zones .
spontaneously , did not exhibit any seizure B Pleural space .
activity , and has no medical history . Her heart C Apical segment of the upper lung lobes .
rate is 90 bpm and blood pressure 110/70 D Cervical or supraclavicular lymph nodes .
mmHg . Neurologic examination is normal . The
pregnancy test is negative . Which of the * A 24 year old man has been treated with INH ,
following is the most appropriate management ? Rifampicine ,and pyrazinamide for active
Admit to hospital for cardiac evaluation . pulmonary tuberculosis . After 3 months , he
Outpatient echocardiogram . states that he is having numbness and tingling of
Twenty four hour Holter monitor . both feet but no Back pain . He denies taking
Reassurance and discharge home . other medications . Which of the following is the
most appropriate next step ?
* A 67 year old woman has diabetes and mild A CT scan of the lumbar spine .
HTN . She is noted to have some diabetic B Initiate pyridoxine .
retinopathy , and she states that she cannot feel C Continue the tuberculosis agents and monitor
her legs . She has recurrent episodes of for further neurological problems .
lightheadedness when she gets up in the D Initiate a workup for tuberculosis Adenopathy
‫ اﺳﺎﻣﻪ اﻟﺨﺰاﻋﻠﻪ‬: ‫اﻟﺪﻛﺘﻮر‬ ‫اﺳﺌﻠــــــــﺔ اﻟﺒﺎﻃﻨﻴــــــﺔ‬
Page 38
E compression on the femoral nerve. A Oligoclonal bands in cerebrospinal fluid .
B Increased N_acetyl aspartate with MR
* A 25 year old woman is seen in the clinic spectroscopy .
because her father was diagnosed with and has C Abnormal peripheral nerve conduction by
been treated for tuberculosis . She denies a electromyograph ( EMG) / nerve conduction
cough and her chest radiograph is normal . A velocity ( NCV) studies .
PPD test shows 10 mm of induration . Her only D Meningeal enhancement on contrast MRI of
medication is an OCP . Which of the following is brain
the best next step ?
A Oral INH and barrier contraception . * A 33 year old man is noted to have
B Combination therapy including INH, exacerbations of weakness . He is diagnosed
Rifampicine and pyrazinamide . with MS . Which of the following findings is
C Observation . consistent with the diagnosis ?
D Induce three sputum samples . A The diagnosis of MS is based on clinical
lesions separated by time and space .
* Which of the following tests is the most B Oligo bands in the CSF are specific for multiple
appropriate to follow for a patient receiving sclerosis .
isoniazed and rifampin for tuberculosis C Steroids are effective in improving the course
treatment of disease .
A Renal function tests D This is a genetic disorder well characterized on
B Liver function tests chromosome 11
C Slit_lamp examinations
D Amylase and lipase tests

* Which of the following is not a dermatologic


manifestation of Dermatomyositis ? * A 33 year old man is noted to have
Calcinosis cutis exacerbations of weakness . He is diagnosed
Malar rash with MS . He is noted to have significantly
Gottron papules progressive disease . Which of the following is
Heliotrope rash likely to be helpful for his symptoms of weakness
A Mitoxantrone
* A 19 year old man develops diarrhea ,and 2 B Corticosteroids therapy
weeks later experiences gait difficulties and foot C Plasmapheresis
tingling , which of the following problems D Immunoglobulin therapy
associated with his condition ?
A Acute inflammatory demyelinating * Which of the following signs is most suggestive
polyneuropathy . of Parkinson disease rather than the other
B Acute stroke . neurodegenerative diseases ?
C Myasthenia gravis . A Unilateral resting tremor .
D Inflammatory Myopathy . B Supranuclear downed gaze palsy .
E Transverse cord myelitis . C Orthostatic hypotension early in the course the
disease .
* A 34 year old woman presents with fatigable D Early falls .
muscle weakness with climbing stairs or blow E Abnormal cerebral MRI .
drying her hair . This is associated with some
shortness of breath , which improves with test . * Which of the following medications is most
Which of the following condition associated with likely to be able to help both relieve cardinal
her symptoms ? features of Parkinson disease as well as reduce
A Acute inflammatory demyelinating drug induced
polyneuropathy . dyskinesias ?
B Acute stroke . A Levodopa
C Myasthenia gravis . B Dopamine agonists
D Inflammatory Myopathy . C Amantadine
E Transverse cord myelitis . D Anticholinergics
E Haloperidol
* Multiple sclerosis is characterized by which of
the following diagnostic findings ?
‫ اﺳﺎﻣﻪ اﻟﺨﺰاﻋﻠﻪ‬: ‫اﻟﺪﻛﺘﻮر‬ ‫اﺳﺌﻠــــــــﺔ اﻟﺒﺎﻃﻨﻴــــــﺔ‬
Page 39
* Which of the following medications would be A . Diltiazem
most likely to cause drug induced B . Metoprolol
Parkinsonism ? C . Coumadin
A Trihexyphenidyl D . Cardioversion
B Metoclopramide E . Aspirin
C Diazepam
D Carbidopa * A 62 year old man is seen in the ED for lower
E Levodopa abdominal pain which has since resolved ,
however , his heart rate was 80 bpm and
* Which of the following tests is used to palpated irregularly . On ECG , he is diagnosed
diagnose benign Paroxysmal positional vertigo ? with atrial fibrillation with a ventricular response
A Weber test of 114 bpm . He does not recall being told about
B Rinne test this condition . Which of the following is the best
C Dix_Hallpike maneuver initial treatment for this patient ?
D Brandt Daroff maneuver A . Diltiazem
E Epley maneuver B . Cardioversion
C . Synthroid
* A 40 year old woman has recurring episodes of D . Ibutilide
disabling vertigo , lasting 30 minutes , and E . Aspirin
accompanied by roaring tinnitus , aural pressure
and low frequency hearing loss . Her physical * Approximately what percentage of patients with
examination is normal . What is the most likely bacterial meningitis present with the classic triad
diagnosis ? of fever , neck stiffness and altered mental
A Benign Paroxysmal positional vertigo . status ?
B Acute suppurative labyrinthitis . A < 25 %
C Acute serous labyrinthitis . B < 50 %
D Meniere disease . C > 75 %
E Vertebrobasilar insufficiency . D > 99 %
* A 45 year old woman complains of recurring
episodes of vertigo that began after she bumped * A 58 year old man presents to his physician's
her head rather forcefully 2 week ago . Her spells office complaining of 2 hours of substernal chest
last 10 to 15 seconds and occur whenever she pain and dyspnea . Which of the following is the
goes to bed or wakes up . She denies any hearing most important next step in management ?
loss or tinnitus . Her physical examination is A . Administration of Propranolol .
normal except for a latent , rotatory nystagmus B . Aspirin to chew .
when she is lying with the right ear down . What C . Sublingual nitroglycerin .
is the most likely diagnosis D . Administration of a diuretic agent .
A . Acute serous labyrinthitis . E . Chest radiograph .
B . Benign Paroxysmal positional vertigo .
C . Vestibular migraine . * A 45 year old man is seen in ED 3 hours of
D . Meniere disease . substernal chest pain radiating to his left arm .
E . Viral vestibular neuronitis . The ECG shows only nonspecific changes .
Hearing that the ECG is normal , he requests to
* A 66 year old woman is noted to have go home . Which of the following statements is
asymptomatic atrial fibrillation. Which of the most accurate ?
following is most common complication of her A . The patient may be safely discharged home .
atrial fibrillation ? B . If a repeat ECG in 30 minutes is normal , MI is
A . sudden death . essentially ruled out and the patient may be
B . stroke . safely discharged .
C . shock . C . The patient should be advised that half of
D . dyspnea . heart attack patients have a nondiagnostic ECG
E . myocardial infarction and serial cardiac biomarkers levels should be
assessed .
* A 65 year old woman is presents to the ED with D . The patient should undergo an immediate
a dyspnea , fatigue , and palpitations . Her blood thallium stress test to further
pressure is 85/50 mmHg and her heart rate is assesses for coronary artery disease to help
140 bpm . Which of the following is the best clarify the management .
treatment for this patient ?
‫ اﺳﺎﻣﻪ اﻟﺨﺰاﻋﻠﻪ‬: ‫اﻟﺪﻛﺘﻮر‬ ‫اﺳﺌﻠــــــــﺔ اﻟﺒﺎﻃﻨﻴــــــﺔ‬
Page 40
* Which of the following tumor markers is
appropriately matched with the cell type cancer * A 24 year old man with a history of poorly
and can be followed during treatment as an treated chronic ulcerative colitis is found to have
adjunct to assess disease burden ? anemia with a hemoglobin of 9 g/dl and a
A CA_125 _ Colon cancer . reduced mean corpuscular volume . His Ferritin
B Calcitonin _ Follicular carcinoma of the thyroid is 250 . Which of the following is the most likely
C CD30 _ Hairy cell leukemia . cause of his anemia ?
D HCG _ GTD . A Folate deficiency
E Neuron specific enolase _ Non small cell B Hemoglobinopathy
carcinoma of the lung C Inflammation
D Iron deficiency
* A 66 year old woman has CLL with a stable E Sideroblastic anemia
white blood cell count of between 60.000 and
70.000 /uL . She is currently hospitalized with * All of the following statements are true
pneumococcal pneumonia . This is the patients regarding the criteria to diagnose
third episode of pneumonia within the past 12 hypereosinophilic syndrome except ?
months . What finding on laboratory testing A . Increased bone marrow Eosinophils must be
would be most likely in this patient ? demonstrated .
A Granulocytopenia . B. It is not necessary to have increased
B Hypogammaglobulinemia . circulating Eosinophils .
C Impaired T cell function with normal T C . Primary myeloid leukemia must be excluded .
lymphocyte count . D . Reactive Eosinophilia must be excluded .
D Low CD4 count . E . There must be less than 20% myeloblasts in
E No specific abnormality is expected . blood or bone marrow .

* A 55 year old man is found to have a serum


calcium of 13 mg/dl after coming to clinic * Which of the following Hemolytic anemias can
complaining of fatigue and thirst for the past be classified as extracorpuscular ?
month . A chest radiograph demonstrates a 4 cm Elliptocytosis
mass in the right lower lobe . Which of the PNH
following serum tests is most likely to reveal the Pyruvate kinase deficiency
cause of his hypercalcemia ? SCA
A . ACTH . TTP
B . ADH .
C . Insulin like growth factor . * Which of the following statements regarding
D . PTH . Polycythemia rubra vera is correct ?
E . PTH related protein . A . And elevated plasma Erythropoietin level
excludes the diagnosis .
* All of the following conditions may be B. Transformation to acute leukemia is
associated with a thymoma except ? common .
A . Erythrocytosis . C . Thrombocytosis correlates strongly with
B .Hypogammaglobulinemia . thrombotic risk .
C . Myasthenia gravis . D . Aspirin should be prescribed to all of these
D . Polymyositis . patients to reduce thrombotic risk .
E . Pure red blood cell aplasia . E . Phlebotomy is used only after hydroxyurea
and interferon have been tried .
* All of the following statements regarding the
anemia of chronic kidney disease are true except * All of the following are vitamin K dependent
A . The degree of anemia correlates with the coagulation factors except ?
stage of chronic kidney disease . A Factor 10
B . Erythropoietin levels are reduced . B Factor 7
C . Ferritin is reduced . C Protein C
D . It is typically Normocytic and normochromic . D Protein S
E . Reticulocytes are decreased . E Factor 8
‫ اﺳﺎﻣﻪ اﻟﺨﺰاﻋﻠﻪ‬: ‫اﻟﺪﻛﺘﻮر‬ ‫اﺳﺌﻠــــــــﺔ اﻟﺒﺎﻃﻨﻴــــــﺔ‬
Page 41
* All of the following laboratory values are
consistent with an intravascular Hemolytic
anemia except ?
A Increased haptoglobin
B Increased LDH
C Increased Reticulocyte count
D Increased unconjugated bilirubin
E Increased urine hemosiderin

* All of the following are consistent with


nonproliferative diabetic retinopathy except ?
A Blot hemorrhages
B Cotton wool spots
C Neovascularization
D Occurs in first or second decade of DM
E Retinal vascular microaneurysms

* All of the following are potential causes of


elevated LDL except ?
A Anorexia nervosa
B Cirrhosis
C Cholestasis
D Nephrotic syndrome
E Thiazide diuretics

* Which of the following is the most common


cause of Cushing syndrome ?
A ACTH producing pituitary adenoma .
B Adrenocortical adenoma .
C Adrenocortical carcinoma .
D Ectopic ACTH secretion .
E McCune Albright syndrome .

* A 27 year old woman with mild obesity is seen


in her primary care office for increased thirst and
Polyuria . DM is suspected , and a random
plasma glucose of 211 mg/d confirms this
diagnosis . Which of the following tests will
strongly indicate that she has type one DM ?
A .Anti GAD 65 antibody .
B .Peroxisome proliferator activated receptor _ 2
polymorphism testing .
C . Plasma insulin level .
D . Testing for HLA_DR3 .
E . There is no laboratory test indicating type one
DM .

‫ اﺳﺎﻣﻪ اﻟﺨﺰاﻋﻠﻪ‬: ‫اﻟﺪﻛﺘﻮر‬ ‫اﺳﺌﻠــــــــﺔ اﻟﺒﺎﻃﻨﻴــــــﺔ‬


Page 42

You might also like